Complex FINAL Review

Pataasin ang iyong marka sa homework at exams ngayon gamit ang Quizwiz!

The nurse weighs a patient daily and measures urinary output every hour. The nurse notices a weight gain of 1.5 kg in a 74-kg patient over 48 hours. The nurse is aware that this weight gain is equivalent to the retention of:

1,500 mL of fluid A 1-kg weight gain is equal to 1,000 mL of retained fluid.

Acute/Intermediate phase

48-72 hours after Red urine suggests hemochromogen from damages RBCs d/t muscle damage Glycosuria is common d/t stress response

Most common sites of Metastasis

Adrenal glands Liver Lungs Bones Brain Lymph nodes

Parkland Burn Formula

4mL x body weight (kg) x percentage of body surface= total fluid requirement in mL for 24 hours to calculate what the IV will run at NOW, take that # and divide it by 2, now divide it by (8-however long it took to get to the ER) to get your hourly IV flow rate half to be given in 1st 8 hours, 1/4 in second 8 hours and 1/4 in last 8 hours...starting at the time of the burn, not arrival to ER

Parkland (Baxter) formula for fluid replacement

4mg/ kg of body weight, and first half within the first 8 hours

A nurse is caring for a patient whose cancer metastasis has resulted in bone pain. Which of the following are typical characteristics of bone pain? A) A dull, deep ache that is boring in nature B) Soreness or aching that may include cramping C) Sharp, piercing pain that is relieved by immobilization D) Spastic or sharp pain that radiates

A

A nurse is providing care for a patient whose pattern of laboratory testing reveals longstanding hypocalcemia. What other laboratory result is most consistent with this finding? A) An elevated parathyroid hormone level B) An increased calcitonin level C) An elevated potassium level D) A decreased vitamin D level

A

A patient has just had an arthroscopy performed to assess a knee injury. What nursing intervention should the nurse implement following this procedure? A) Wrap the joint in a compression dressing. B) Perform passive range of motion exercises. C) Maintain the knee in flexion for up to 30 minutes. D) Apply heat to the knee.

A

A patients fracture is healing and callus is being deposited in the bone matrix. This process characterizes what phase of the bone healing process? A) The reparative phase B) The reactive phase C) The remodeling phase D) The revascularization phase

A

A patient has had a total mastectomy with immediate reconstruction. The patient asks the nurse when she can take a shower. What should the nurse respond? A) Not until the drain is removed B) On the second postoperative day C) Now, if you wash gently with soap and water D) Seven days after your surgery

A (Feedback: If immediate reconstruction has been performed, showering may be contraindicated until the drain is removed.)

A patient who just suffered a suspected ischemic stroke is brought to the ED by ambulance. On what should the nurse's primary assessment focus? A) Cardiac and respiratory status B) Seizure activity C) Pain D) Fluid and electrolyte balance

A (Feedback: Acute care begins with managing ABCs. Patients may have difficulty keeping an open and clear airway secondary to decreased LOC. Neurological assessment with close monitoring for signs of increased neurologic deficit and seizure activity occurs next. Fluid and electrolyte balance must be controlled carefully with the goal of adequate hydration to promote perfusion and decrease further brain activity.)

Following an addisonian crisis, a patients adrenal function has been gradually regained. The nurse should ensure that the patient knows about the need for supplementary glucocorticoid therapy in which of the following circumstances? A) Episodes of high psychosocial stress B) Periods of dehydration C) Episodes of physical exertion D) Administration of a vaccine

A Feedback: During stressful procedures or significant illnesses, additional supplementary therapy with glucocorticoids is required to prevent addisonian crisis. Physical activity, dehydration and vaccine administration would not normally be sufficiently demanding such to require glucocorticoids.

The nurse who is leading a wellness workshop has been asked about actions to reduce the risk of bladder cancer. What health promotion action most directly addresses a major risk factor for bladder cancer? A) Smoking cessation B) Reduction of alcohol intake C) Maintenance of a diet high in vitamins and nutrients D) Vitamin D supplementation

A Feedback: People who smoke develop bladder cancer twice as often as those who do not smoke. High alcohol intake and low vitamin intake are not noted to contribute to bladder cancer.

A public health nurse is teaching a health class for the male students at the local high school. The nurse is teaching the boys to perform monthly testicular self-examinations. What point would be appropriate to emphasize? A) Testicular cancer is a highly curable type of cancer. B) Testicular cancer is very difficult to diagnose. C) Testicular cancer is the number one cause of cancer deaths in males. D) Testicular cancer is more common in older men.

A Feedback: Testicular cancer is highly curable, particularly when it's treated in its early stage. Self-examination allows early detection and facilitates the early initiation of treatment. The highest mortality rates from cancer among men are with lung cancer. Testicular cancer is found more commonly in younger men.

A nurse is teaching a 53-year-old man about prostate cancer. What information should the nurse provide to best facilitate the early identification of prostate cancer? A) Have a digital rectal examination and prostate-specific antigen (PSA) test done yearly. B) Have a transrectal ultrasound every 5 years. C) Perform monthly testicular self-examinations, especially after age 60. D) Have a complete blood count (CBC), blood urea nitrogen (BUN) and creatinine assessment performed annually.

A Feedback: The incidence of prostate cancer increases after age 50. The digital rectal examination, which identifies enlargement or irregularity of the prostate, and the PSA test, a tumor marker for prostate cancer, are effective diagnostic measures that should be done yearly. Testicular self-examinations won't identify changes in the prostate gland due to its location in the body. A transrectal ultrasound and CBC with BUN and creatinine assessment are usually done after diagnosis to identify the extent of disease and potential metastases.

You are developing a care plan for a patient with Cushing syndrome. What nursing diagnosis would have the highest priority in this care plan? A) Risk for injury related to weakness B) Ineffective breathing pattern related to muscle weakness C) Risk for loneliness related to disturbed body image D) Autonomic dysreflexia related to neurologic changes

A Feedback: The nursing priority is to decrease the risk of injury by establishing a protective environment. The patient who is weak may require assistance from the nurse in ambulating to prevent falls or bumping corners or furniture. The patients breathing will not be affected and autonomic dysreflexia is not a plausible risk. Loneliness may or may not be an issue for the patient, but safety is a priority.

A child is growing at a rate appropriate for his age. What cells are responsible for the secretion of bone matrix that eventually results in bone growth? A) Osteoblasts B) Osteocytes C) Osteoclasts D) Lamellae

A 'blasts' for BONE growth; like a rocket! 'cytes' are BONE cells for maintenance 'clasts' are BONE reabsorption/destroyers

A nurse has been called for duty during a response to a natural disaster. In this context of care, the nurse should expect to do which of the following? A) Practice outside of her normal area of clinical expertise. B) Perform interventions that are not based on assessment data. C) Prioritize psychosocial needs over physiologic needs. D) Prioritize the interests of older adults over younger patients.

A During a disaster, nurses may be asked to perform duties outside their areas of expertise and may take on responsibilities normally held by physicians or advanced practice nurses.

An 83-year-old patient is brought in by ambulance from a long-term care facility. The patients symptoms are weakness, lethargy, incontinence, and a change in mental status. The nurse knows that emergencies in older adults may be more difficult to manage. Why would this be true? A) Older adults may have an altered response to treatment. B) Older adults are often reluctant to adhere to prescribed treatment. C) Older adults have difficulty giving a health history. D) Older adults often stigmatize their peers who use the ED.

A Emergencies in this age group may be more difficult to manage because elderly patients may have an atypical presentation, an altered response to treatment, a greater risk of developing complications, or a combination of these factors. The elderly patient may perceive the emergency as a crisis signaling the end of an independent lifestyle or even resulting in death. Stigmatization and nonadherence to treatment are not commonly noted. Older adults do not necessarily have difficulty giving a health history.

A nurse takes a shift report and finds he is caring for a patient who has been exposed to anthrax by inhalation. What precautions does the nurse know must be put in place when providing care for this patient? A) Standard precautions B) Airborne precautions C) Droplet precautions D) Contact precautions

A The patient is not contagious, and anthrax cannot be spread from person to person, so standard precautions are initiated. Airborne, contact, and droplet precautions are not necessary.

The nurse in a pediatric ICU is caring for a child who is dying of sickle cell anemia. The child's mother has been unable to eat or sleep and can talk only about her impending loss and the guilt she feels about the child's pain and suffering. What intervention has the highest priority? A) Allowing the patient to express her feelings without judging her B) Helping the patient to understand the phases of the grieving process C) Reassuring the patient that the child's death is not her fault D) Arranging for genetic counseling to inform the patient of her chances of having another child with the disease

A) Allowing the patient to express her feelings without judging her Listening to the patient express her feelings openly without judging her is the highest priority. The nurse should not impose his or her own values on the patient. The nurse should also help the patient to understand the grieving process and use all the support systems that are available to assist her in coping with this situation. Genetic counseling may be appropriate at a later time.

You are caring for a 50-year-old man diagnosed with multiple myeloma; he has just been told by the care team that his prognosis is poor. He is tearful and trying to express his feelings, but he is having difficulty. What should you do first? A) Ask if he would like you to sit with him while he collects his thoughts. B) Tell him that you will leave for now but will be back shortly. C) Offer to call pastoral care or a member of his chosen clergy. D) Reassure him that you can understand how he is feeling.

A) Ask if he would like you to sit with him while he collects his thoughts. The most important intervention the nurse can provide is listening empathetically. Seriously ill patients and their families need time and support to cope with the changes brought about by serious illness and the prospect of impending death. The nurse who is able to listen without judging and without trying to solve the patient's and family's problems provides an invaluable intervention. The patient needs to feel that people are concerned with his situation. Leaving him does not show acceptance of his feelings. Offering to call pastoral care may be helpful for some patients, but should be done after you have spent time with the patient. Telling the patient that you understand how he is feeling is inappropriate because it does not help him express his feelings.

A nurse is caring for a patient with Hodgkin lymphoma at the oncology clinic. The nurse should be aware of what main goal of care? A) Cure of the disease B) Enhancing quality of life C) Controlling symptoms D) Palliation

A) Cure of the disease The goal in the treatment of Hodgkin lymphoma is cure. Palliation is thus not normally necessary. Quality of life and symptom control are vital, but the overarching goal is the cure the disease.

A nurse who provides care on an acute medical unit has observed that physicians are frequently reluctant to refer patients to hospice care. What are contributing factors that are known to underlie this tendency? Select all that apply. A) Financial pressures on health care providers B) Patient reluctance to accept this type of care C) Strong association of hospice care with prolonging death D) Advances in "curative" treatment in late-stage illness E) Ease of making a terminal diagnosis

A) Financial pressures on health care providers B) Patient reluctance to accept this type of care D) Advances in "curative" treatment in late-stage illness Physicians are reluctant to refer patients to hospice, and patients are reluctant to accept this form of care. Reasons include the difficulties in making a terminal prognosis (especially for those patients with noncancer diagnoses), the strong association of hospice with death, advances in "curative" treatment options in late-stage illness, and financial pressures on health care providers that may cause them to retain rather than refer hospice-eligible patients.

An oncology nurse is caring for a patient with multiple myeloma who is experiencing bone destruction. When reviewing the patient's most recent blood tests, the nurse should anticipate what imbalance? A) Hypercalcemia B) Hyperproteinemia C) Elevated serum viscosity D) Elevated RBC count

A) Hypercalcemia Hypercalcemia may result when bone destruction occurs due to the disease process. Elevated serum viscosity occurs because plasma cells excrete excess immunoglobulin. RBC count will be decreased. Hyperproteinemia would not be present.

A nurse is caring for a patient who is postoperative day 1 right hip replacement. How should the nurse position the patient? A) Keep the patients hips in abduction at all times. B) Keep hips flexed at no less than 90 degrees. C) Elevate the head of the bed to high Fowler's. D) Seat the patient in a low chair as soon as possible.

A) Keep the patients hips in abduction at all times. (If the legs go inward 'adduction' then the hip can be popped out of socket...visualize this.) The hips should be kept in abduction by an abductor pillow. Hips should not be flexed more than 90 degrees, and the head of bed should not be elevated more than 60 degrees. The patients hips should be higher than the knees; as such, high seat chairs should be used.

The nurse educator on an orthopedic trauma unit is reviewing the safe and effective use of traction with some recent nursing graduates. What principle should the educator promote? A) Knots in the rope should not be resting against pulleys. B) Weights should rest against the bed rails. C) The end of the limb in traction should be braced by the footboard of the bed. D) Skeletal traction may be removed for brief periods to facilitate the patients independence.

A) Knots in the rope should not be resting against pulleys. Knots in the rope should not rest against pulleys, because this interferes with traction. Weights are used to apply the vector of force necessary to achieve effective traction and should hang freely at all times. To avoid interrupting traction, the limb in traction should not rest against anything. Skeletal traction is never interrupted.

A nurse is caring for a patient who has a diagnosis of acute leukemia. What assessment most directly addresses the most common cause of death among patients with leukemia? A) Monitoring for infection B) Monitoring nutritional status C) Monitor electrolyte levels D) Monitoring liver function

A) Monitoring for infection In patients with acute leukemia, death typically occurs from infection or bleeding. Compromised nutrition, electrolyte imbalances, and impaired liver function are all plausible, but none is among the most common causes of death in this patient population.

A nurse is admitting a patient to the unit who presented with a lower extremity fracture. What signs and symptoms would suggest to the nurse that the patient may have a peroneal nerve injury? A) Numbness and burning of the foot B) Pallor to the dorsal surface of the foot C) Visible cyanosis in the toes D) Inadequate capillary refill to the toes

A) Numbness and burning of the foot Peroneal nerve injury may result in numbness, tingling, and burning in the feet. Cyanosis, pallor, and decreased capillary refill are signs of inadequate circulation.

A patient with a diagnosis of acute myeloid leukemia (AML) is being treated with induction therapy on the oncology unit. What nursing action should be prioritized in the patient's care plan? A) Protective isolation and vigilant use of standard precautions B) Provision of a high-calorie, low-texture diet and appropriate oral hygiene C) Including the family in planning the patient's activities of daily living D) Monitoring and treating the patient's pain

A) Protective isolation and vigilant use of standard precautions Induction therapy causes neutropenia and a severe risk of infection. This risk must be addressed directly in order to ensure the patient's survival. For this reason, infection control would be prioritized over nutritional interventions, family care, and pain, even though each of these are important aspects of nursing care.

An oncology nurse is providing health education for a patient who has recently been diagnosed with leukemia. What should the nurse explain about commonalities between all of the different subtypes of leukemia? A) The different leukemias all involve unregulated proliferation of white blood cells. B) The different leukemias all have unregulated proliferation of red blood cells and decreased bone marrow function. C) The different leukemias all result in a decrease in the production of white blood cells. D) The different leukemias all involve the development of cancer in the lymphatic system.

A) The different leukemias all involve unregulated proliferation of white blood cells. Leukemia commonly involves unregulated proliferation of white blood cells. Decreased production of red blood cells is associated with anemias. Decreased production of white blood cells is associated with leukopenia. The leukemias are not characterized by their involvement with the lymphatic system.

A 60-year-old patient with chronic myeloid leukemia will be treated in the home setting and the nurse is preparing appropriate health education. What topic should the nurse emphasize? A) The importance of adhering to the prescribed drug regimen B) The need to ensure that vaccinations are up to date C) The importance of daily physical activity D) The need to avoid shellfish and raw foods

A) The importance of adhering to the prescribed drug regimen Nurses need to understand that the effectiveness of the drugs used to treat CML is based on the ability of the patient to adhere to the medication regimen as prescribed. Adherence is often incomplete, thus this must be a focus of health education. Vaccinations normally would not be administered during treatment and daily physical activity may be impossible for the patient. Dietary restrictions are not normally necessary.

An adult oncology patient has a diagnosis of bladder cancer with metastasis and the patient has asked the nurse about the possibility of hospice care. Which principle is central to a hospice setting? A) The patient and family should be viewed as a single unit of care. B) Persistent symptoms of terminal illness should not be treated. C) Each member of the interdisciplinary team should develop an individual plan of care. D) Terminally ill patients should die in the hospital whenever possible.

A) The patient and family should be viewed as a single unit of care. Hospice care requires that the patient and family be viewed as a single unit of care. The other listed principles are wholly inconsistent with the principles of hospice care.

A patient with suspected adrenal insufficiency has been ordered an adrenocorticotropic hormone(ACTH) stimulation test. Administration of ACTH caused a marked increase in cortisol levels. How should the nurse interpret this finding? A) The patients pituitary function is compromised. B) The patients adrenal insufficiency is not treatable. C) The patient has insufficient hypothalamic function. D) The patient would benefit from surgery.

A) The patients pituitary function is compromised.

Following a recent history of dyspareunia and lower abdominal pain, a patient has received a diagnosis of pelvic inflammatory disease (PID). When providing health education related to self-care, the nurse should address which of the following topics? Select all that apply. A) Use of condoms to prevent infecting others B) Appropriate use of antibiotics C) Taking measures to prevent pregnancy D) The need for a Pap smear every 3 months E) The importance of weight loss in preventing symptoms

A) Use of condoms to prevent infecting others B) Appropriate use of antibiotics

The nurse is developing a plan of care for a patient with Guillain-Barr syndrome. Which of the following interventions should the nurse prioritize for this patient? A) Using the incentive spirometer as prescribed B) Maintaining the patient on bed rest C) Providing aids to compensate for loss of vision D) Assessing frequently for loss of cognitive function

A) Using the incentive spirometer as prescribed Have intubation equipment available at bedside

Following an extensive diagnostic workup, an older adult patient has been diagnosed with a secondary myelodysplastic syndrome (MDS). What assessment question most directly addresses the potential etiology of this patient's health problem? A) Were you ever exposed to toxic chemicals in any of the jobs that you held? B) When you were younger, did you tend to have recurrent infections of any kind? C) Have your parents or siblings had any disease like this? D) Would you say that you've had a lot of sun exposure in your lifetime?"

A) Were you ever exposed to toxic chemicals in any of the jobs that you held? Secondary MDS can occur at any age and results from prior toxic exposure to chemicals, including chemotherapeutic medications. Family history, sun exposure, and previous infections are unrelated to the pathophysiology of secondary MDS.

A nurse is explaining a patients decreasing bone density in terms of the balance between bone resorption and formation. What dietary nutrients and hormones play a role in the resorption and formation of adult bones? Select all that apply. A) Thyroid hormone B) Growth hormone C) Estrogen D) Vitamin B12 E) Luteinizing hormone

A, B, C

The nurse is assessing a patient for dietary factors that may influence her risk for osteoporosis. The nurse should question the patient about her intake of what nutrients? Select all that apply. A) Calcium B) Simple carbohydrates C) Vitamin D D) Protein E) Soluble fiber

A, C

A patient with Cushing syndrome has been hospitalized after a fall. The dietician consulted works with the patient to improve the patients nutritional intake. What foods should a patient with Cushing syndrome eat to optimize health? Select all that apply. A) Foods high in vitamin D B) Foods high in calories C) Foods high in protein D) Foods high in calcium E) Foods high in sodium

A, C, D Feedback: Foods high in vitamin D, protein, and calcium are recommended to minimize muscle wasting and osteoporosis. Referral to a dietitian may assist the patient in selecting appropriate foods that are also low in sodium and calories.

A patient with trichomoniasis comes to the walk-in clinic. In developing a care plan for this patient the nurse would know to include what as an important aspect of treating this patient? A) Both partners will be treated with metronidazole (Flagyl). B) Constipation and menstrual difficulties may occur. C) The patient should perform Kegel exercises 30 to 80 times daily. D) Care will involve hormone therapy to control the pain.

A. The most effective treatment for trichomoniasis is metronidazole (Flagyl). Both partners receive a one-time loading dose or a smaller dose three times a day for 1 week. In pelvic inflammatory disease, menstrual difficulties and constipation may occur. Kegel exercises are prescribed to help strengthen weakened muscles associated with cystocele and other structural deficits. Hormone therapy does not address the etiology of trichomoniasis.

cranial nerve six VI

Abducens

ABCDE

Airway Breathing Circulation Disability Environment

A nurse is assessing a patient with an acoustic neuroma who has been recently admitted to an oncology unit. What symptoms is the nurse likely to find during the initial assessment? A) Loss of hearing, tinnitus, and vertigo B) Loss of vision, change in mental status, and hyperthermia C) Loss of hearing, increased sodium retention, and hypertension D) Loss of vision, headache, and tachycardia

Ans: A Feedback: An acoustic neuroma is a tumor of the eighth cranial nerve, the cranial nerve most responsible for hearing and balance. The patient with an acoustic neuroma usually experiences loss of hearing, tinnitus, and episodes of vertigo and staggering gait. Acoustic neuromas do not cause loss of vision, increased sodium retention, or tachycardia.

A 50-year-old man diagnosed with leukemia will begin chemotherapy. What would the nurse do to combat the most common adverse effects of chemotherapy? A) Administer an antiemetic. B) Administer an antimetabolite. C) Administer a tumor antibiotic. D) Administer an anticoagulant.

Ans: A Feedback: Antiemetics are used to treat nausea and vomiting, the most common adverse effects of chemotherapy. Antihistamines and certain steroids are also used to treat nausea and vomiting. Antimetabolites and tumor antibiotics are classes of chemotherapeutic medications. Anticoagulants slow blood clotting time, thereby helping to prevent thrombi and emboli.

A nurse is planning the care of an older adult patient with osteomalacia. What action should the nurse recommend in order to promote vitamin D synthesis? A) Ensuring adequate exposure to sunlight B) Eating a low-purine diet C) Performing cardiovascular exercise while avoiding weight-bearing exercises D) Taking thyroid supplements as ordered

Ans: A Feedback: Because sunlight is necessary for synthesizing vitamin D, patients should be encouraged to spend some time in the sun. A low-purine diet is not a relevant action and thyroid supplements do not directly affect bone function. Action must be taken to prevent fractures, but weight-bearing exercise within safe parameters is not necessarily contraindicated.

A patient with Huntington disease has just been admitted to a long-term care facility. The charge nurse is creating a care plan for this patient. Nutritional management for a patient with Huntington disease should be informed by what principle? A) The patient is likely to have an increased appetite. B) The patient is likely to required enzyme supplements. C) The patient will likely require a clear liquid diet. D) The patient will benefit from a low-protein diet.

Ans: A Feedback: Due to the continuous involuntary movements, patients will have a ravenous appetite. Despite this ravenous appetite, patients usually become emaciated and exhausted. As the disease progresses, patients experience difficulty in swallowing and thin liquids should be avoided. Protein will not be limited with this disease. Enzyme supplements are not normally required.

The nurse has created a plan of care for a patient who is at risk for increased ICP. The patients care plan should specify monitoring for what early sign of increased ICP? A) Disorientation and restlessness (anxiety) B) Decreased pulse and respirations C) Projectile vomiting D) Loss of corneal reflex

Ans: A Feedback: Early indicators of ICP include disorientation and restlessness. Later signs include decreased pulse and respirations, projectile vomiting, and loss of brain stem reflexes, such as the corneal reflex.

A patient is undergoing diagnostic testing to determine the etiology of recent joint pain. The patient asks the nurse about the difference between osteoarthritis (OA) and rheumatoid arthritis (RA). What is the best response by the nurse? A) OA is a considered a noninflammatory joint disease. RA is characterized by inflamed, swollen joints. B) OA and RA are very similar. OA affects the smaller joints such as the fingers, and RA affects the larger, weight-bearing joints like the knees. C) OA originates with an infection. RA is a result of your body's cells attacking one another. D) OA is associated with impaired immune function; RA is a consequence of physical damage.

Ans: A Feedback: OA is a degenerative arthritis with a noninflammatory etiology, characterized by the loss of cartilage on the articular surfaces of weight-bearing joints, with spur development. RA is characterized by inflammation of synovial membranes and surrounding structures. The diseases are not distinguished by the joints affected and neither has an infectious etiology.

A patient with a new diagnosis of amyotrophic lateral sclerosis (ALS) is overwhelmed by his diagnosis and the known complications of the disease. How can the patient best make known his wishes for care as his disease progresses? A) Prepare an advance directive. B) Designate a most responsible physician (MRP) early in the course of the disease. C) Collaborate with representatives from the Amyotrophic Lateral Sclerosis Association. D) Ensure that witnesses are present when he provides instruction.

Ans: A Feedback: Patients with ALS are encouraged to complete an advance directive or living will to preserve their autonomy in decision making. None of the other listed actions constitutes a legally binding statement of end-of-life care.

A patient who has been newly diagnosed with systemic lupus erythematosus (SLE) has been admitted to the medical unit. Which of the following nursing diagnoses is the most plausible inclusion in the plan of care? A) Fatigue Related to Anemia B) Risk for Ineffective Tissue Perfusion Related to Venous Thromboembolism C) Acute Confusion Related to Increased Serum Ammonia Levels D) Risk for Ineffective Tissue Perfusion Related to Increased Hematocrit

Ans: A Feedback: Patients with SLE nearly always experience fatigue, which is partly attributable to anemia. Ammonia levels are not affected and hematocrit is typically low, not high. VTE is not one of the central complications of SLE.

A 16-year-old female patient experiences alopecia resulting from chemotherapy, prompting the nursing diagnosis of disturbed body image and situational low self-esteem. What action by the patient would best indicate that she is meeting the goal of improved body image and self-esteem? A) The patient requests that her family bring her makeup and wig. B) The patient begins to discuss the future with her family. C) The patient reports less disruption from pain and discomfort. D) The patient cries openly when discussing her disease.

Ans: A Feedback: Requesting her wig and makeup indicates that the patient with alopecia is becoming interested in looking her best and that her body image and self-esteem may be improving. The other options may indicate that other nursing goals are being met, but they do not necessarily indicate improved body image and self-esteem.

A patient presents to a clinic complaining of a leg ulcer that isn't healing; subsequent diagnostic testing suggests osteomyelitis. The nurse is aware that the most common pathogen to cause osteomyelitis is what? A) Staphylococcus aureus B) Proteus C) Pseudomonas D) Escherichia coli

Ans: A Feedback: S. aureus causes over 50% of bone infections. Proteus, Pseudomonas, and E. coli are also causes, but to a lesser extent.

The home health nurse is performing a home visit for an oncology patient discharged 3 days ago after completing treatment for non-Hodgkin lymphoma. The nurses assessment should include examination for the signs and symptoms of what complication? A) Tumor lysis syndrome (TLS) B) Syndrome of inappropriate antidiuretic hormone (SIADH) C) Disseminated intravascular coagulation (DIC) D) Hypercalcemia

Ans: A Feedback: TLS is a potentially fatal complication that occurs spontaneously or more commonly following radiation, biotherapy, or chemotherapy-induced cell destruction of large or rapidly growing cancers such as leukemia, lymphoma, and small cell lung cancer. DIC, SIADH and hypercalcemia are less likely complications following this treatment and diagnosis.

The nurse is caring for a patient with permanent neurologic impairments resulting from a traumatic head injury. When working with this patient and family, what mutual goal should be prioritized? A) Achieve as high a level of function as possible. B) Enhance the quantity of the patients life. C) Teach the family proper care of the patient. D) Provide community assistance.

Ans: A Feedback: The overarching goals of care are to achieve as high a level of function as possible and to enhance the quality of life for the patient with neurologic impairment and his or her family. This goal encompasses family and community participation.

A patient with suspected Parkinson's disease is initially being assessed by the nurse. When is the best time to assess for the presence of a tremor? A) When the patient is resting B) When the patient is ambulating C) When the patient is preparing his or her meal tray to eat D) When the patient is participating in occupational therapy

Ans: A Feedback: The tremor is present while the patient is at rest; it increases when the patient is walking, concentrating, or feeling anxious. Resting tremor characteristically disappears with purposeful movement, but is evident when the extremities are motionless. Consequently, the nurse should assess for the presence of a tremor when the patient is not performing deliberate actions.

A nurse is working with a patient with rheumatic disease who is being treated with salicylate therapy. What statement would indicate that the patient is experiencing adverse effects of this drug? A) I have this ringing in my ears that just wont go away. B) I feel so foggy in the mornings and it takes me so long to wake up. C) When I eat a meal that's high in fat, I get really nauseous. D) I seem to have lost my appetite, which is unusual for me.

Ans: A Feedback: Tinnitus is associated with salicylate therapy. Salicylates do not normally cause drowsiness, intolerance of high-fat meals, or anorexia.

A 60-year-old patient with a diagnosis of prostate cancer is scheduled to have an interstitial implant for high-dose radiation (HDR). What safety measure should the nurse include in this patients subsequent plan of care? A) Limit the time that visitors spend at the patients bedside. B) Teach the patient to perform all aspects of basic care independently. C) Assign male nurses to the patients care whenever possible. D) Situate the patient in a shared room with other patients receiving brachytherapy.

Ans: A Feedback: To limit radiation exposure, visitors should generally not spend more than 30 minutes with the patient. Pregnant nurses or visitors should not be near the patient, but there is no reason to limit care to nurses who are male. All necessary care should be provided to the patient and a single room should be used.

An 82-year-old man is admitted for observation after a fall. Due to his age, the nurse knows that the patient is at increased risk for what complication of his injury? A) Hematoma B) Skull fracture C) Embolus D) Stroke

Ans: A Feedback: Two major factors place older adults at increased risk for hematomas. First, the dura becomes more adherent to the skull with increasing age. Second, many older adults take aspirin and anticoagulants as part of routine management of chronic conditions. Because of these factors, the patients risk of a hematoma is likely greater than that of stroke, embolism, or skull fracture.

The nurse is planning discharge education for a patient with trigeminal neuralgia. The nurse knows to include information about factors that precipitate an attack. What would the nurse be correct in teaching the patient to avoid? A) Washing his face B) Exposing his skin to sunlight C) Using artificial tears D) Drinking large amounts of fluids

Ans: A Feedback: Washing the face should be avoided if possible because this activity can trigger an attack of pain in a patient with trigeminal neuralgia. Using artificial tears would be an appropriate behavior. Exposing the skin to sunlight would not be harmful to this patient. Temperature extremes in beverages should be avoided.

You are caring for a patient who has just been told that her stage IV colon cancer has recurred and metastasized to the liver. The oncologist offers the patient the option of surgery to treat the progression of this disease. What type of surgery does the oncologist offer? A) Palliative B) Reconstructive C) Salvage D) Prophylactic

Ans: A Feedback: When cure is not possible, the goals of treatment are to make the patient as comfortable as possible and to promote quality of life as defined by the patient and his or her family. Palliative surgery is performed in an attempt to relieve complications of cancer, such as ulceration, obstruction, hemorrhage, pain, and malignant effusion. Reconstructive surgery may follow curative or radical surgery in an attempt to improve function or obtain a more desirable cosmetic effect. Salvage surgery is an additional treatment option that uses an extensive surgical approach to treat the local recurrence of a cancer after the use of a less extensive primary approach. Prophylactic surgery involves removing nonvital tissues or organs that are at increased risk to develop cancer. Test Bank - Brunner & Suddarth's Textbook of Medical-Surgical Nursing 14e (Hinkle 2017) 313

A patients decreased mobility is ultimately the result of an autoimmune reaction originating in the synovial tissue, which caused the formation of pannus. This patient has been diagnosed with what health problem? A) Rheumatoid arthritis (RA) B) Systemic lupus erythematosus C) Osteoporosis D) Polymyositis

Ans: A Morning joint stiffness lasting more than 1 hour. Feedback: In RA, the autoimmune reaction results in phagocytosis, producing enzymes within the joint that break down collagen, cause edema and proliferation of the synovial membrane, and ultimately form pannus. Pannus destroys cartilage and bone. SLE, osteoporosis, and polymyositis do not involve pannus formation.

5. A hospital patient is immunocompromised because of stage 3 HIV infection and the physician has ordered a chest radiograph. How should the nurse most safely facilitate the test? A) Arrange for a portable x-ray machine to be used. B) Have the patient wear a mask to the x-ray department. C) Ensure that the radiology department has been disinfected prior to the test. D) Send the patient to the x-ray department, and have the staff in the department wear masks.

Ans: A Feedback: A patient who is immunocompromised is at an increased risk of contracting nosocomial infections due to suppressed immunity. The safest way the test can be facilitated is to have a portable x-ray machine in the patients room. This confers more protection than disinfecting the radiology department or using masks.

8. A nurse is planning the care of a patient with AIDS who is admitted to the unit withPneumocystis pneumonia (PCP). Which nursing diagnosis has the highest priority for this patient? A) Ineffective Airway Clearance B) Impaired Oral Mucous Membranes C) Imbalanced Nutrition: Less than Body Requirements D) Activity Intolerance

Ans: A Feedback: Although all these nursing diagnoses are appropriate for a patient with AIDS, Ineffective Airway Clearance is the priority nursing diagnosis for the patient with Pneumocystis pneumonia (PCP). Airway and breathing take top priority over the other listed concerns.

2. A clinic nurse is caring for a patient admitted with AIDS. The nurse has assessed that the patient is experiencing a progressive decline in cognitive, behavioral, and motor functions. The nurse recognizes that these symptoms are most likely related to the onset of what complication? A) HIV encephalopathy B) B-cell lymphoma C) Kaposis sarcoma D) Wasting syndrome

Ans: A Feedback: HIV encephalopathy is a clinical syndrome characterized by a progressive decline in cognitive, behavioral, and motor functions. The other listed complications do not normally have cognitive and behavioral manifestations.

The health care team is caring for a patient with osteomalacia. It has been determined that the osteomalacia is caused by malabsorption. What is the usual treatment for osteomalacia caused by malabsorption? A) Supplemental calcium and increased doses of vitamin D B) Exogenous parathyroid hormone and multivitamins C) Colony-stimulating factors and calcitonin D) Supplemental potassium and pancreatic enzymes

Ans: A Feedback: If osteomalacia is caused by malabsorption, increased doses of vitamin D, along with supplemental calcium, are usually prescribed.

22. A nurse is addressing the incidence and prevalence of HIV infection among older adults. What principle should guide the nurses choice of educational interventions? A) Many older adults do not see themselves as being at risk for HIV infection. B) Many older adults are not aware of the difference between HIV and AIDS. C) Older adults tend to have more sex partners than younger adults. D) Older adults have the highest incidence of intravenous drug use.

Ans: A Feedback: It is known that many older adults do not see themselves as being at risk for HIV infection. Knowledge of the relationship between HIV infection and AIDS is not known to affect the incidence of new cases. The statements about sex partners and IV drug use are untrue.

34. A nurse is assessing the skin integrity of a patient who has AIDS. When performing this inspection, the nurse should prioritize assessment of what skin surfaces? A) Perianal region and oral mucosa B) Sacral region and lower abdomen C) Scalp and skin over the scapulae D) Axillae and upper thorax

Ans: A Feedback: The nurse should inspect all the patients skin surfaces and mucous membranes, but the oral mucosa and perianal region are particularly vulnerable to skin breakdown and fungal infection. Test Bank - Brunner & Suddarth's Textbook of Medical-Surgical Nursing 14e (Hinkle 2017) 708

18. A nurse is caring for a patient in the emergent/resuscitative phase of burn injury. During this phase, the nurse should monitor for evidence of what alteration in laboratory values? A) Sodium deficit B) Decreased prothrombin time (PT) C) Potassium deficit D) Decreased hematocrit

Ans: A Feedback: Anticipated fluid and electrolyte changes that occur during the emergent/resuscitative phase of burn injury include sodium deficit, potassium excess, base-bicarbonate deficit, and elevated hematocrit. PT does not typically decrease.

35. A burn patient is transitioning from the acute phase of the injury to the rehabilitation phase. The patient tells the nurse, I cant wait to have surgery to reconstruct my face so I look normal again. What would be the nurses best response? A) That's something that you and your doctor will likely talk about after your scars mature. B) That is something for you to talk to your doctor about because it's not a nursing responsibility. C) I know this is really important to you, but you have to realize that no one can make you look like you used to. D) Unfortunately, it's likely that you will have most of these scars for the rest of your life.

Ans: A Feedback: Burn reconstruction is a treatment option after all scars have matured and is discussed within the first few years after injury. Even though this is not a nursing responsibility, the nurse should still respond appropriately to the patients query. It is true that the patient will not realistically look like he or she used to, but this does not instill hope.

3. A patient in the emergent/resuscitative phase of a burn injury has had blood work and arterial blood gases drawn. Upon analysis of the patients laboratory studies, the nurse will expect the results to indicate what? A) Hyperkalemia, hyponatremia, elevated hematocrit, and metabolic acidosis B) Hypokalemia, hypernatremia, decreased hematocrit, and metabolic acidosis C) Hyperkalemia, hypernatremia, decreased hematocrit, and metabolic alkalosis D) Hypokalemia, hyponatremia, elevated hematocrit, and metabolic alkalosis

Ans: A Feedback: Fluid and electrolyte changes in the emergent/resuscitative phase of a burn injury include hyperkalemia related to the release of potassium into the extracellular fluid, hyponatremia from large amounts of sodium lost in trapped edema fluid, hemoconcentration that leads to an increased hematocrit, and loss of bicarbonate ions that results in metabolic acidosis.

A nurse is caring for a critically ill patient with autonomic dysreflexia. What clinical manifestations would the nurse expect in this patient? A) Respiratory distress and projectile vomiting B) Bradycardia and hypertension C) Tachycardia and agitation D) Third-spacing and hyperthermia

Ans: B Feedback: Autonomic dysreflexia is characterized by a pounding headache, profuse sweating, nasal congestion, piloerection (goose bumps), bradycardia, and hypertension. It occurs in cord lesions above T6 after spinal shock has resolved; it does not result in vomiting, tachycardia, or third-spacing.

8. A patient has sustained a severe burn injury and is thought to have an impaired intestinal mucosal barrier. Since this patient is considered at an increased risk for infection, what intervention will best assist in avoiding increased intestinal permeability and prevent early endotoxin translocation? A) Early enteral feeding B) Administration of prophylactic antibiotics C) Bowel cleansing procedures D) Administration of stool softeners

Ans: A Feedback: If the intestinal mucosa receives some type of protection against permeability change, infection could be avoided. Early enteral feeding is one step to help avoid this increased intestinal permeability and prevent early endotoxin translocation. Antibiotics are seldom prescribed prophylactically because of the risk of promoting resistant strains of bacteria. A bowel cleansing procedure would not be ordered for this patient. The administration of stool softeners would not assist in avoiding increased intestinal permeability and prevent early endotoxin translocation.

An 80-year-old man in a long-term care facility has a chronic leg ulcer and states that the area has become increasingly painful in recent days. The nurse notes that the site is now swollen and warm to the touch. The patient should undergo diagnostic testing for what health problem? A) Osteomyelitis B) Osteoporosis C) Osteomalacia D) Septic arthritis

Ans: A Feedback: When osteomyelitis develops from the spread of an adjacent infection, no signs of septicemia are present, but the area becomes swollen, warm, painful, and tender to touch. Osteoporosis is the most prevalent bone disease in the world. Osteomalacia is a metabolic bone disease characterized by inadequate mineralization of bone. Septicarthritis occurs when joints become infected through spread of infection from other parts of the body (hematogenous spread) or directly through trauma or surgical instrumentation.

A patient has been admitted to the medical unit for the treatment of Pagets disease. When reviewing the medication administration record, the nurse should anticipate what medications? Select all that apply. A) Calcitonin B) Bisphosphonates C) Alkaline phosphatase D) Calcium gluconate E) Estrogen

Ans: A, B Feedback: Bisphosphonates are the cornerstone of Paget therapy in that they stabilize the rapid bone turnover. Calcitonin is also used because it retards bone resorption by decreasing the number and availability of osteoclasts. Alkaline phosphatase is a naturally occurring enzyme, not a drug. Calcium gluconate and estrogen are not used in the treatment of Pagets disease.

A patient with spinal cord injury is ready to be discharged home. A family member asks the nurse to review potential complications one more time. What are the potential complications that should be monitored for in this patient? Select all that apply. A) Orthostatic hypotension B) Autonomic dysreflexia C) DVT D) Salt-wasting syndrome E) Increased ICP

Ans: A, B, C Feedback: For a spinal cord-injured patient, based on the assessment data, potential complications that may develop include DVT, orthostatic hypotension, and autonomic dysreflexia. Salt-wasting syndrome or increased ICP are not typical complications following the immediate recovery period.

The nurse is describing some of the major characteristics of cancer to a patient who has recently received a diagnosis of malignant melanoma. When differentiating between benign and malignant cancer cells, the nurse should explain differences in which of the following aspects? Select all that apply. A) Rate of growth B) Ability to cause death C) Size of cells D) Cell contents E) Ability to spread

Ans: A, B, E Feedback: Benign and malignant cells differ in many cellular growth characteristics, including the method and rate of growth, ability to metastasize or spread, general effects, destruction of tissue, and ability to cause death. Cells come in many sizes, both benign and malignant. Cell contents are basically the same, but they behave differently.

You are caring for an adult patient who has developed a mild oral yeast infection following chemotherapy. What actions should you encourage the patient to perform? Select all that apply. A) Use a lip lubricant. B) Scrub the tongue with a firm-bristled toothbrush. C) Use dental floss every 24 hours. D) Rinse the mouth with normal saline. E) Eat spicy food to aid in eradicating the yeast.

Ans: A, C, D Feedback: Stomatitis is an inflammation of the oral cavity. The patient should be encouraged to brush the teeth with a soft toothbrush after meals, use dental floss every 24 hours, rinse with normal saline, and use a lip lubricant. Mouthwashes and hot foods should be avoided.

A male patient presents to the clinic complaining of a headache. The nurse notes that the patient is guarding his neck and tells the nurse that he has stiffness in the neck area. The nurse suspects the patient may have meningitis. What is another well-recognized sign of this infection? A) Negative Brudzinski's sign B) Positive Kernig's sign C) Hyper Patellar reflex D) Sluggish pupil reaction

Ans: B Feedback: Meningeal irritation results in a number of well-recognized signs commonly seen in meningitis, such as a positive Kernig's sign, a positive Brudzinski's sign, and photophobia. Hyper Patellar reflex and a sluggish pupil reaction are not commonly recognized signs of meningitis.

A patient with rheumatoid arthritis comes to the clinic complaining of pain in the joint of his right great toe and is eventually diagnosed with gout. When planning teaching for this patient, what management technique should the nurse emphasize? A) Take OTC calcium supplements consistently. B) Restrict consumption of foods high in purines. C) Ensure fluid intake of at least 4 liters per day. D) Restrict weight-bearing on right foot.

Ans: B Feedback: Although severe dietary restriction is not necessary, the nurse should encourage the patient to restrict consumption of foods high in purines, especially organ meats. Calcium supplementation is not necessary and activity should be maintained as tolerated. Increased fluid intake is beneficial, but it is not necessary for the patient to consume more than 4 liters daily. (sweetbread, aged cheeses and meats)

A nurse is performing the initial assessment of a patient who has a recent diagnosis of systemic lupus erythematosus (SLE). What skin manifestation would the nurse expect to observe on inspection? A) Petechiae B) Butterfly rash C) Jaundice D) Skin sloughing

Ans: B Feedback: An acute cutaneous lesion consisting of a butterfly-shaped rash across the bridge of the nose and cheeks occurs in SLE. Petechiae are pinpoint skin hemorrhages, which are not a clinical manifestation of SLE. Patients with SLE do not typically experience jaundice or skin sloughing.

The nurse is developing a plan of care for a patient newly diagnosed with Bells palsy. The nurses plan of care should address what characteristic manifestation of this disease? A) Tinnitus B) Facial paralysis C) Pain at the base of the tongue D) Diplopia

Ans: B Feedback: Bells palsy is characterized by facial dysfunction, weakness, and paralysis. It does not result in diplopia, pain at the base of the tongue, or tinnitus.

A patient with possible bacterial meningitis is admitted to the ICU. What assessment finding would the nurse expect for a patient with this diagnosis? A) Pain upon ankle dorsiflexion of the foot B) Neck flexion produces flexion of knees and hips C) Inability to stand with eyes closed and arms extended without swaying D) Numbness and tingling in the lower extremities

Ans: B Feedback: Clinical manifestations of bacterial meningitis include a positive Brudzinskis sign. Neck flexion producing flexion of knees and hips correlates with a positive Brudzinskis sign. Positive Homans sign (pain upon dorsiflexion of the foot) and negative Rombergs sign (inability to stand with eyes closed and arms extended) are not expected assessment findings for the patient with bacterial meningitis. Peripheral neuropathy manifests as numbness and tingling in the lower extremities. Again, this would not be an initial assessment to rule out bacterial meningitis.

A patients most recent diagnostic imaging has revealed that his lung cancer has metastasized to his bones and liver. What is the most likely mechanism by which the patients cancer cells spread? A) Hematologic spread B) Lymphatic circulation C) Invasion D) Angiogenesis

Ans: B Feedback: Lymph and blood are key mechanisms by which cancer cells spread. Lymphatic spread (the transport of Test Bank - Brunner & Suddarth's Textbook of Medical-Surgical Nursing 14e (Hinkle 2017) 309 tumor cells through the lymphatic circulation) is the most common mechanism of metastasis.

A nurse is creating a plan of care for an oncology patient and one of the identified nursing diagnosis is risk for infection related to myelosuppression. What intervention addresses the leading cause of infection-related death in oncology patients? A) Encourage several small meals daily. B) Provide skin care to maintain skin integrity. C) Assist the patient with hygiene, as needed. D) Assess the integrity of the patients oral mucosa regularly.

Ans: B Feedback: Nursing care for patients with skin reactions includes maintaining skin integrity, cleansing the skin, promoting comfort, reducing pain, preventing additional trauma, and preventing and managing infection. Malnutrition in oncology patients may be present, but it is not the leading cause of infection-related death. Poor hygiene does not normally cause events that result in death. Broken oral mucosa may be an avenue for infection, but it is not the leading cause of death in an oncology patient.

A patient has come to the clinic for a routine annual physical. The nurse practitioner notes a palpable, painless projection of bone at the patients shoulder. The projection appears to be at the distal end of the humerus. The nurse should suspect the presence of which of the following? A) Osteomyelitis B) Osteochondroma C) Osteomalacia D) Pagets disease

Ans: B Feedback: Osteochondroma is the most common benign bone tumor. It usually occurs as a large projection of bone at the end of long bones (at the knee or shoulder). Osteomyelitis, osteomalacia, and Pagets disease do not involve the development of excess bone tissue.

The nurse is caring for a boy who has muscular dystrophy. When planning assistance with the patients ADLs, what goal should the nurse prioritize? A) Promoting the patients recovery from the disease B) Maximizing the patients level of function C) Ensuring the patients adherence to treatment D) Fostering the family's participation in care

Ans: B Feedback: Priority for the care of the child with muscular dystrophy is the need to maximize the patients level of function. Family participation is also important, but should be guided by this goal. Adherence is not a central goal, even though it is highly beneficial, and the disease is not curable.

The nurse is caring for a patient with an advanced stage of breast cancer and the patient has recently learned that her cancer has metastasized. The nurse enters the room and finds the patient struggling to breath and the nurses rapid assessment reveals that the patients jugular veins are distended. The nurse should suspect the development of what oncologic emergency? A) Increased intracranial pressure B) Superior vena cava syndrome (SVCS) C) Spinal cord compression D) Metastatic tumor of the neck

Ans: B Feedback: SVCS occurs when there is gradual or sudden impaired venous drainage giving rise to progressive shortness of breath (dyspnea), cough, hoarseness, chest pain, and facial swelling; edema of the neck, arms, hands, and thorax and reported sensation of skin tightness and difficulty swallowing; as well as possibly engorged and distended jugular, temporal, and arm veins. Increased intracranial pressure may be a part of SVCS, but it is not what is causing the patients symptoms. The scenario does not mention a problem with the patients spinal cord. The scenario says that the cancer has metastasized, but not that it has metastasized to the neck.

An oncology nurse educator is providing health education to a patient who has been diagnosed with skin cancer. The patients wife has asked about the differences between normal cells and cancer cells. What characteristic of a cancer cell should the educator site? A) Malignant cells contain more fibronectin than normal body cells. B) Malignant cells contain proteins called tumor-specific antigens. C) Chromosomes contained in cancer cells are more durable and stable than those of normal cells. D) The nuclei of cancer cells are unusually large, but regularly shaped.

Ans: B Feedback: The cell membranes are altered in cancer cells, which affect fluid movement in and out of the cell. The cell membrane of malignant cells also contains proteins called tumor-specific antigens. Malignant cellular membranes also contain less fibronectin, a cellular cement. Typically, nuclei of cancer cells are large and irregularly shaped (pleomorphism). Fragility of chromosomes is commonly found when cancer cells are analyzed.

35. A hospital nurse has experienced percutaneous exposure to an HIV-positive patients blood as a result of a needlestick injury. The nurse has informed the supervisor and identified the patient. What action should the nurse take next? A) Flush the wound site with chlorhexidine. B) Report to the emergency department or employee health department. C) Apply a hydrocolloid dressing to the wound site. D) Follow up with the nurses primary care provider.

Ans: B Feedback: After initiating the emergency reporting system, the nurse should report as quickly as possible to the employee health services, the emergency department, or other designated treatment facility. Flushing is recommended, but chlorhexidine is not used for this purpose. Applying a dressing is not recommended. Following up with the nurses own primary care provider would require an unacceptable delay.

9. A public health nurse is preparing an educational campaign to address a recent local increase in the incidence of HIV infection. The nurse should prioritize which of the following interventions? A) Lifestyle actions that improve immune function B) Educational programs that focus on control and prevention C) Appropriate use of standard precautions D) Screening programs for youth and young adults

Ans: B Feedback: Until an effective vaccine is developed, preventing HIV by eliminating and reducing risk behaviors is essential. Educational interventions are the primary means by which behaviors can be influenced. Screening is appropriate, but education is paramount. Enhancing immune function does not prevent HIV infection. Ineffective use of standard precautions apply to very few cases of HIV infection. Test Bank - Brunner & Suddarth's Textbook of Medical-Surgical Nursing 14e (Hinkle 2017) 697

10. A nurse is working with a patient who was diagnosed with HIV several months earlier. The nurse should recognize that a patient with HIV is considered to have AIDS at the point when the CD4+ T-lymphocyte cell count drops below what threshold? A) 75 cells/mm3 of blood B) 200 cells/mm3 of blood C) 325 cells/mm3 of blood D) 450 cells/mm3 of blood

Ans: B Feedback: When CD4+ T-cell levels drop below 200 cells/mm3 of blood, the person is said to have AIDS.

4. A patient has experienced an electrical burn and has developed thick eschar over the burn site. Which of the following topical antibacterial agents will the nurse expect the physician to order for the wound? A) Silver sulfadiazine 1% (Silvadene) water-soluble cream B) Mafenide acetate 10% (Sulfamylon) hydrophilic-based cream C) Silver nitrate 0.5% aqueous solution D) Acticoat

Ans: B Feedback: Mafenide acetate 10% hydrophilic-based cream is the agent of choice when there is a need to penetrate thick eschar. Silver products do not penetrate eschar; Acticoat is a type of silver dressing.

The nurse on a bone marrow transplant unit is caring for a patient with cancer who is preparing for HSCT. What is a priority nursing diagnosis for this patient? A) Fatigue related to altered metabolic processes B) Altered nutrition: less than body requirements related to anorexia C) Risk for infection related to altered immunologic response D) Body image disturbance related to weight loss and anorexia

Ans: C Feedback: A priority nursing diagnosis for this patient is risk for infection related to altered immunologic response. Because the patients immunity is suppressed, he or she will be at a high risk for infection. The other listed nursing diagnoses are valid, but they are not as high a priority as is risk for infection.

The ED nurse is caring for a patient who has been brought in by ambulance after sustaining a fall at home. What physical assessment finding is suggestive of a basilar skull fracture? A) Epistaxis B) Periorbital edema C) Bruising over the mastoid D) Unilateral facial numbness

Ans: C Feedback: An area of ecchymosis (bruising) may be seen over the mastoid) in a basilar skull fracture. Battles sign Numbness, edema, and epistaxis are not directly associated with a basilar skull fracture.

A nurse is reviewing the pathophysiology that may underlie a patients decreased bone density. What hormone should the nurse identify as inhibiting bone resorption and promoting bone formation? A) Estrogen B) Parathyroid hormone (PTH) C) Calcitonin D) Progesterone

Ans: C Feedback: Calcitonin inhibits bone resorption and promotes bone formation, estrogen inhibits bone breakdown, and parathyroid increases bone resorption. Estrogen, which inhibits bone breakdown, decreases with aging. Parathyroid hormone (PTH) increases with aging, increasing bone turnover and resorption. Progesterone is the major naturally occurring human progestogen and plays a role in the female menstrual cycle.

A patient is being admitted to the neurologic ICU with suspected herpes simplex virus encephalitis. What nursing action best addresses the patients complaints of headache? A) Initiating a patient-controlled analgesia (PCA) of morphine sulfate B) Administering hydromorphone (Dilaudid) IV as needed C) Dimming the lights and reducing stimulation D) Distracting the patient with activity

Ans: C Feedback: Comfort measures to reduce headache include dimming the lights, limiting noise and visitors, grouping nursing interventions, and administering analgesic agents. Opioid analgesic medications may mask neurologic symptoms; therefore, they are used cautiously. Non-opioid analgesics may be preferred. Distraction is unlikely to be effective, and may exacerbate the patients pain.

A patient with increased ICP has a ventriculostomy for monitoring ICP. The nurses most recent assessment reveals that the patient is now exhibiting nuchal rigidity and photophobia. The nurse would be correct in suspecting the presence of what complication? A) Encephalitis B) CSF leak C) Meningitis D) Catheter occlusion

Ans: C Feedback: Complications of a ventriculostomy include ventricular infectious meningitis and problems with the monitoring system. Nuchal rigidity and photophobia are clinical manifestations of meningitis, but are not suggestive of encephalitis, a CSF leak, or an occluded catheter.

A clinic nurse is caring for a patient with suspected gout. While explaining the pathophysiology of gout to the patient, the nurse should describe which of the following? A) Autoimmune processes in the joints B) Chronic metabolic acidosis C) Increased uric acid levels D) Unstable serum calcium levels

Ans: C Feedback: Gout is caused by hyperuricemia (increased serum uric acid). Gout is not categorized as an autoimmune disease and it does not result from metabolic acidosis or unstable serum calcium levels.

A patient, brought to the clinic by his wife and son, is diagnosed with Huntington disease. When providing anticipatory guidance, the nurse should address the future possibility of what effect of Huntington disease? A) Metastasis B) Risk for stroke C) Emotional and personality changes D) Pathologic bone fractures

Ans: C Feedback: Huntington disease causes profound changes to personality and behavior. It is a nonmalignant disease and stroke is not a central risk. The disease is not associated with pathologic bone fractures.

A patient with amyotrophic lateral sclerosis (ALS) is being visited by the home health nurse who is creating a care plan. What nursing diagnosis is most likely for a patient with this condition? A) Chronic confusion B) Impaired urinary elimination C) Impaired verbal communication D) Bowel incontinence

Ans: C Feedback: Impaired communication is an appropriate nursing diagnosis; the voice in patients with ALS assumes a nasal sound and articulation becomes so disrupted that speech is unintelligible. Intellectual function is marginally impaired in patients with late ALS. Usually, the anal and bladder sphincters are intact Test Bank - Brunner & Suddarth's Textbook of Medical-Surgical Nursing 14e (Hinkle 2017) 1322 because the spinal nerves that control muscles of the rectum and urinary bladder are not affected.

A nurse is performing the health history and physical assessment of a patient who has a diagnosis of rheumatoid arthritis (RA). What assessment finding is most consistent with the clinical presentation of RA? A) Cool joints with decreased range of motion B) Signs of systemic infection C) Joint stiffness, especially in the morning D) Visible atrophy of the knee and shoulder joints

Ans: C Feedback: In addition to joint pain and swelling, another classic sign of RA is joint stiffness, especially in the morning. Joints are typically swollen, not atrophied, and systemic infection does not accompany the disease. Joints are often warm rather than cool.

A 33-year-old patient presents at the clinic with complaints of weakness, incoordination, dizziness, and loss of balance. The patient is hospitalized and diagnosed with MS. What sign or symptom, revealed during the initial assessment, is typical of MS? A) Diplopia, history of increased fatigue, and decreased or absent deep tendon reflexes B) Flexor spasm, clonus, and negative Babinski's reflex C) Blurred vision, intention tremor, and urinary hesitancy D) Hyperactive abdominal reflexes and history of unsteady gait and episodic paresthesia in both legs

Ans: C Feedback: Optic neuritis, leading to blurred vision, is a common early sign of MS, as is intention tremor (tremor when performing an activity). Nerve damage can cause urinary hesitancy. In MS, deep tendon reflexes are increased or hyperactive. A positive Babinskis reflex is found in MS. Abdominal reflexes are absent with MS.

An oncology patient has just returned from the postanesthesia care unit after an open hemicolectomy. This patients plan of nursing care should prioritize which of the following? A) Assess the patient hourly for signs of compartment syndrome. B) Assess the patients fine motor skills once per shift. C) Assess the patients wound for dehiscence every 4 hours. D) Maintain the patients head of bed at 45 degrees or more at all times.

Ans: C Feedback: Postoperatively, the nurse assesses the patients responses to the surgery and monitors the patient for possible complications, such as infection, bleeding, thrombophlebitis, wound dehiscence, fluid and electrolyte imbalance, and organ dysfunction. Fine motor skills are unlikely to be affected by surgery and compartment syndrome is a complication of fracture casting, not abdominal surgery. There is no need to maintain a high head of bed.

Traditionally, nurses have been involved with tertiary cancer prevention. However, an increasing emphasis is being placed on both primary and secondary prevention. What would be an example of primary prevention? A) Yearly Pap tests B) Testicular self-examination C) Teaching patients to wear sunscreen D) Screening mammograms

Ans: C Feedback: Primary prevention is concerned with reducing the risks of cancer in healthy people through practices such as use of sunscreen. Secondary prevention involves detection and screening to achieve early diagnosis, as demonstrated by Pap tests, mammograms, and testicular exams.

The nurse is caring for a 39-year-old woman with a family history of breast cancer. She requested a breast tumor marking test and the results have come back positive. As a result, the patient is requesting a bilateral mastectomy. This surgery is an example of what type of oncologic surgery? A) Salvage surgery B) Palliative surgery C) Prophylactic surgery D) Reconstructive surgery

Ans: C Feedback: Prophylactic surgery is used when there is an extensive family history and nonvital tissues are removed. Salvage surgery is an additional treatment option that uses an extensive surgical approach to treat the local recurrence of a cancer after the use of a less extensive primary approach. Palliative surgery is performed in an attempt to relieve complications of cancer, such as ulceration, obstruction, hemorrhage, pain, and malignant effusion. Reconstructive surgery may follow curative or radical surgery in an attempt to improve function or obtain a more desirable cosmetic effect.

A public health nurse has formed an interdisciplinary team that is developing an educational program entitled Cancer: The Risks and What You Can Do About Them. Participants will receive information, but the major focus will be screening for relevant cancers. This program is an example of what type of health promotion activity? A) Disease prophylaxis B) Risk reduction C) Secondary prevention D) Tertiary prevention

Ans: C Feedback: Secondary prevention involves screening and early detection activities that seek to identify early stage cancer in individuals who lack signs and symptoms suggestive of cancer. Primary prevention is concerned with reducing the risks of disease through health promotion strategies. Tertiary prevention is the care and rehabilitation of the patient after having been diagnosed with cancer.

The hospice nurse is caring for a patient with cancer in her home. The nurse has explained to the patient and the family that the patient is at risk for hypercalcemia and has educated them on that signs and symptoms of this health problem. What else should the nurse teach this patient and family to do to reduce the patients risk of hypercalcemia? A) Stool softeners are contraindicated. B) Laxatives should be taken daily. C) Consume 2 to 4 L of fluid daily. D) Restrict calcium intake.

Ans: C Feedback: The nurse should identify patients at risk for hypercalcemia, assess for signs and symptoms of hypercalcemia, and educate the patient and family. The nurse should teach at-risk patients to recognize and report signs and symptoms of hypercalcemia and encourage patients to consume 2 to 4 L of fluid daily unless contraindicated by existing renal or cardiac disease. Also, the nurse should explain the use Test Bank - Brunner & Suddarth's Textbook of Medical-Surgical Nursing 14e (Hinkle 2017) 314 of dietary and pharmacologic interventions, such as stool softeners and laxatives for constipation, and advise patients to maintain nutritional intake without restricting normal calcium intake.

A patient is being admitted to the neurologic ICU following an acute head injury that has resulted in cerebral edema. When planning this patients care, the nurse would expect to administer what priority medication? A) Hydrochlorothiazide (HydroDIURIL) B) Furosemide (Lasix) C) Mannitol (Osmitrol) D) Spirolactone (Aldactone)

Ans: C Feedback: The osmotic diuretic mannitol is given to dehydrate the brain tissue and reduce cerebral edema. This drug acts by reducing the volume of brain and extracellular fluid. Spironolactone, furosemide, and hydrochlorothiazide are diuretics that are not typically used in the treatment of increased ICP resulting from cerebral edema.

A 58-year-old male patient has been hospitalized for a wedge resection of the left lower lung lobe after a routine chest x-ray shows carcinoma. The patient is anxious and asks if he can smoke. Which statement by the nurse would be most therapeutic? A) Smoking is the reason you are here. B) The doctor left orders for you not to smoke. C) You are anxious about the surgery. Do you see smoking as helping? D) Smoking is OK right now, but after your surgery it is contraindicated.

Ans: C Test Bank - Brunner & Suddarth's Textbook of Medical-Surgical Nursing 14e (Hinkle 2017) 308 Feedback: Stating You are anxious about the surgery. Do you see smoking as helping? acknowledges the patients feelings and encourages him to assess his previous behavior. Saying Smoking is the reason you are here belittles the patient. Citing the doctors orders does not address the patients anxiety. Sanctioning smoking would be highly detrimental to this patient.

18. A nurse is caring for a patient hospitalized with AIDS. A friend comes to visit the patient and privately asks the nurse about the risk of contracting HIV when visiting the patient. What is the nurses best response? A) Do you think that you might already have HIV? B) Dont worry. Your immune system is likely very healthy. C) AIDS isnt transmitted by casual contact. D) You cant contract AIDS in a hospital setting.

Ans: C Feedback: AIDS is commonly transmitted by contact with blood and body fluids. Patients, family, and friends must be reassured that HIV is not spread through casual contact. A healthy immune system is not necessarily a protection against HIV. A hospital setting does not necessarily preclude HIV infection.

20. A nurse would identify that a colleague needs additional instruction on standard precautions when the colleague exhibits which of the following behaviors? A) The nurse wears face protection, gloves, and a gown when irrigating a wound. B) The nurse washes the hands with a waterless antiseptic agent after removing a pair of soiled gloves. C) The nurse puts on a second pair of gloves over soiled gloves while performing a bloody procedure. D) The nurse places a used needle and syringe in the puncture-resistant container without capping the needle.

Ans: C Feedback: Gloves must be changed after contact with materials that may contain high concentration of microorganisms, even when working with the same patient. Each of the other listed actions adheres to standard precautions.

6. The mother of two young children has been diagnosed with HIV and expresses fear of dying. How should the nurse best respond to the patient? A) Would you like me to have the chaplain come speak with you? B) Youll learn much about the promise of a cure for HIV. C) Can you tell me what concerns you most about dying? D) You need to maintain hope because you may live for several years.

Ans: C Feedback: The nurse can help the patient verbalize feelings and identify resources for support. The nurse should respond with an open-ended question to help the patient to identify fears about being diagnosed with a life-threatening chronic illness. Immediate deferral to spiritual care is not a substitute for engaging with the patient. The nurse should attempt to foster hope, but not in a way that downplays the patients expressed fears.

A nurse is creating a teaching plan for a patient who has a recent diagnosis of scleroderma. What topics should the nurse address during health education? Select all that apply. A) Surgical treatment options B) The importance of weight loss C) Managing Raynaud's-type symptoms D) Smoking cessation E) The importance of vigilant skin care

Ans: C, D, E Feedback: Patient teaching for the patient with scleroderma focuses on management of Raynauds phenomenon, smoking cessation, and meticulous skin care. Surgical treatment options do not exist and weight loss is not a central concern.

A nurse is providing care for a patient who has a recent diagnosis of giant cell arteritis (GCA). What aspect of physical assessment should the nurse prioritize? A) Assessment for subtle signs of bleeding disorders B) Assessment of the metatarsal joints and phalangeal joints C) Assessment for thoracic pain that is exacerbated by activity D) Assessment for headaches and jaw pain

Ans: D AKA Arteritis Feedback: Assessment of the patient with GCA focuses on musculoskeletal tenderness, weakness, and decreased function. Careful attention should be directed toward assessing the head (for changes in vision, headaches, and jaw claudication). There is not a particular clinical focus on the potential for bleeding, hand and foot pain, or thoracic pain.

A patient who suffered a spinal cord injury is experiencing an exaggerated autonomic response. What aspect of the patients current health status is most likely to have precipitated this event? A) The patient received a blood transfusion. B) The patients analgesia regimen was recent changed. C) The patient was not repositioned during the night shift. D) The patients urinary catheter became occluded.

Ans: D Feedback: A distended bladder is the most common cause of autonomic dysreflexia. Infrequent positioning is a less likely cause, although pressure ulcers or tactile stimulation can cause it. Changes in mediations or blood transfusions are unlikely causes.

A patient is suspected of having rheumatoid arthritis and her diagnostic regimen includes aspiration of synovial fluid from the knee for a definitive diagnosis. The nurse knows that which of the following procedures will be involved? A) Angiography B) Myelography C) Paracentesis D) Arthrocentesis

Ans: D Feedback: Arthrocentesis involves needle aspiration of synovial fluid. Angiography is an x-ray study of circulation with a contrast agent injected into a selected artery. Myelography is an x-ray of the spinal subarachnoid space taken after the injection of a contrast agent into the spinal subarachnoid space through a lumbar puncture. Paracentesis is removal of fluid (ascites) from the peritoneal cavity through a small surgical incision or puncture made through the abdominal wall under sterile conditions.

A patients rheumatoid arthritis (RA) has failed to respond appreciably to first-line treatments and the primary care provider has added prednisone to the patients drug regimen. What principle will guide this aspect of the patients treatment? A) The patient will need daily blood testing for the duration of treatment. B) The patient must stop all other drugs 72 hours before starting prednisone. C) The drug should be used at the highest dose the patient can tolerate. D) The drug should be used for as short a time as possible.

Ans: D Feedback: Corticosteroids are used for shortest duration and at lowest dose possible to minimize adverse effects. Daily blood work is not necessary and the patient does not need to stop other drugs prior to using corticosteroids.

What should the nurse suspect when hourly assessment of urine output on a patient post-craniotomy exhibits a urine output from a catheter of 1,500 mL for two consecutive hours? A) Cushing syndrome B) Syndrome of inappropriate antidiuretic hormone (SIADH) C) Adrenal crisis D) Diabetes insipidus

Ans: D Feedback: Diabetes insipidus is an abrupt onset of extreme polyuria that commonly occurs in patients after brain surgery. Cushing syndrome is excessive glucocorticoid secretion resulting in sodium and water retention. SIADH is the result of increased secretion of ADH; the patient becomes volume-overloaded, urine output diminishes, and serum sodium concentration becomes dilute. Adrenal crisis is under secretion of glucocorticoids resulting in profound hypoglycemia, hypovolemia, and hypotension. Monitor output.

The nurse is caring for a patient has just been given a 6-month prognosis following a diagnosis of extensive stage small-cell lung cancer. The patient states that he would like to die at home, but the team believes that the patients care needs are unable to be met in a home environment. What might you suggest as an alternative? A) Discuss a referral for rehabilitation hospital. B) Panel the patient for a personal care home. C) Discuss a referral for acute care. D) Discuss a referral for hospice care.

Ans: D Feedback: Hospice care can be provided in several settings. Because of the high cost associated with free-standing hospices, care is often delivered by coordinating services provided by both hospitals and the community. The primary goal of hospice care is to provide support to the patient and family. Patients who are referred to hospice care generally have fewer than 6 months to live. Each of the other listed options would be less appropriate for the patients physical and psychosocial needs.

A patient is undergoing diagnostic testing for osteomalacia. Which of the following laboratory results is most suggestive of this diagnosis? A) High chloride, calcium, and magnesium B) High parathyroid and calcitonin levels C) Low serum calcium and magnesium levels D) Low serum calcium and low phosphorus level

Ans: D Feedback: Laboratory studies will reveal a low serum calcium and low phosphorus level.

A patient, diagnosed with cancer of the lung, has just been told he has metastases to the brain. What change in health status would the nurse attribute to the patients metastatic brain disease? A) Chronic pain B) Respiratory distress C) Fixed pupils D) Personality changes

Ans: D Feedback: Neurologic signs and symptoms include headache, gait disturbances, visual impairment, personality changes, altered mentation (memory loss and confusion), focal weakness, paralysis, aphasia, and seizures. Pain, respiratory distress, and fixed pupils are not among the more common neurologic signs and symptoms of metastatic brain disease.

An ED nurse has just received a call from EMS that they are transporting a 17-year-old man who has just sustained a spinal cord injury (SCI). The nurse recognizes that the most common cause of this type of injury is what? A) Sports-related injuries B) Acts of violence C) Injuries due to a fall D) Motor vehicle accidents

Ans: D Feedback: The most common causes of SCIs are motor vehicle crashes (46%), falls (22%), violence (16%), and sports (12%).

A nurse is providing a class on osteoporosis at the local seniors center. Which of the following statements related to osteoporosis is most accurate? A) Osteoporosis is categorized as a disease of the elderly. B) A non modifiable risk factor for osteoporosis is a persons level of activity. C) Secondary osteoporosis occurs in women after menopause. D) Slow discontinuation of corticosteroid therapy can halt the progression of the osteoporosis.

Ans: D Feedback: When corticosteroid therapy is discontinued, the progression of osteoporosis is halted, but restoration of lost bone mass does not occur. Osteoporosis is not a disease of the elderly because its onset occurs earlier in life, when bone mass peaks and then begins to decline. A persons level of physical activity is a modifiable factor that influences peak bone mass. Lack of activity increases the risk for the development of osteoporosis. Primary osteoporosis occurs in women after menopause.

37. A patient has been diagnosed with AIDS complicated by chronic diarrhea. What nursing intervention would be appropriate for this patient? A) Position the patient in the high Fowler's position whenever possible. B) Temporarily eliminate animal protein from the patients diet. C) Make sure the patient eats at least two servings of raw fruit each day. D) Obtain a stool culture to identify possible pathogens.

Ans: D Feedback: A stool culture should be obtained to determine the possible presence of microorganisms that cause diarrhea. Patients should generally avoid raw fruit when having diarrhea. There is no need to avoid animal protein or increase the height of the patients bed.

21. An 18-year-old pregnant female has tested positive for HIV and asks the nurse if her baby is going to be born with HIV. What is the nurses best response? A) There is no way to know that for certain, but we do know that your baby has a one in four chance of being born with HIV. B) Your physician is likely the best one to ask that question. C) If the baby is HIV positive there is nothing that can be done until it is born, so try your best not to worry about it now. D) Its possible that your baby could contract HIV, either before, during, or after delivery.

Ans: D Feedback: Mother-to-child transmission of HIV-1 is possible and may occur in utero, at the time of delivery, or through breast-feeding. There is no evidence that the infants risk is 25%. Deferral to the physician is not a substitute for responding appropriately to the patients concern. Downplaying the patients concerns is inappropriate.

23. A 16-year-old has come to the clinic and asks to talk to a nurse. The nurse asks the teen what she needs and the teen responds that she has become sexually active and is concerned about getting HIV. The teen asks the nurse what she can do keep from getting HIV. What would be the nurses best response? A) Theres no way to be sure you wont get HIV except to use condoms correctly. B) Only the correct use of a female condom protects against the transmission of HIV. C) There are new ways of protecting yourself from HIV that are being discovered every day. D) Other than abstinence, only the consistent and correct use of condoms is effective in preventing HIV.

Ans: D Feedback: Other than abstinence, consistent and correct use of condoms is the only effective method to decrease the risk of sexual transmission of HIV infection. Both female and male condoms confer significant protection. New prevention techniques are not commonly discovered, though advances in treatment are constant.

7. The nurse is addressing condom use in the context of a health promotion workshop. When discussing the correct use of condoms, what should the nurse tell the attendees? A) Attach the condom prior to erection. B) A condom may be reused with the same partner if ejaculation has not occurred. C) Use skin lotion as a lubricant if alternatives are unavailable. D) Hold the condom by the cuff upon withdrawal.

Ans: D Feedback: The condom should be unrolled over the hard penis before any kind of sex. The condom should be held by the tip to squeeze out air. Skin lotions, baby oil, petroleum jelly, or cold cream should not be used with condoms because they cause latex deterioration/condom breakage. The condom should be held during withdrawal so it does not come off the penis. Condoms should never be reused.

A patient is brought to the emergency department from the site of a chemical fire, where he suffered a burn that involves the epidermis, dermis, and the muscle and bone of the right arm. On inspection, the skin appears charred. Based on these assessment findings, what is the depth of the burn on the patients arm? A) Superficial partial-thickness B) Deep partial-thickness C) Full partial-thickness D) Full-thickness

Ans: D Feedback: A full-thickness burn involves total destruction of the epidermis and dermis and, in some cases, underlying tissue as well. Wound color ranges widely from white to red, brown, or black. The burned area is painless because the nerve fibers are destroyed. The wound can appear leathery; hair follicles and sweat glands are destroyed. Edema may also be present. Superficial partial-thickness burns involve the epidermis and possibly a portion of the dermis; the patient will experience pain that is soothed by cooling. Deep partial-thickness burns involve the epidermis, upper dermis, and portion of the deeper dermis; the patient will complain of pain and sensitivity to cold air. Full partial thickness is not a depth of burn.

26. A patients burns are estimated at 36% of total body surface area; fluid resuscitation has been ordered in the emergency department. After establishing intravenous access, the nurse should anticipate the administration of what fluid? A) 0.45% NaCl with 20 mEq/L KCl B) 0.45% NaCl with 40 mEq/L KCl C) Normal saline D) Lactated Ringers

Ans: D Feedback: Fluid resuscitation with lactated Ringers (LR) should be initiated using the American Burn Associations (ABA) fluid resuscitation formulas. LR is the crystalloid of choice because its composition and osmolality most closely resemble plasma and because use of normal saline is associated with hyperchloremic acidosis. Potassium chloride solutions would exacerbate the hyperkalemia that occurs following burn injuries.

5. An occupational health nurse is called to the floor of a factory where a worker has sustained a flash burn to the right arm. The nurse arrives and the flames have been extinguished. The next step is to cool the burn. How should the nurse cool the burn? A) Apply ice to the site of the burn for 5 to 10 minutes. B) Wrap the patients affected extremity in ice until help arrives. C) Apply an oil-based substance or butter to the burned area until help arrives. D) Wrap cool towels around the affected extremity intermittently.

Ans: D Feedback: Once the burn has been sustained, the application of cool water is the best first-aid measure. Soaking the burn area intermittently in cool water or applying cool towels gives immediate and striking relief from pain, and limits local tissue edema and damage. However, never apply ice directly to the burn, never wrap the person in ice, and never use cold soaks or dressings for longer than several minutes; such procedures may worsen the tissue damage and lead to hypothermia in people with large burns. Butter is contraindicated.

13. A nurse is caring for a patient who has sustained a deep partial-thickness burn injury. In prioritizing the nursing diagnoses for the plan of care, the nurse will give the highest priority to what nursing diagnosis? A) Activity Intolerance B) Anxiety C) Ineffective Coping D) Acute Pain

Ans: D Feedback: Pain is inevitable during recovery from any burn injury. Pain in the burn patient has been described as one of the most severe causes of acute pain. Management of the often-severe pain is one of the most difficult challenges facing the burn team. While the other nursing diagnoses listed are valid, the presence of pain may contribute to these diagnoses. Management of the patients pain is the priority, as it may have a direct correlation to the other listed nursing diagnoses.

16. A patient is brought to the ED by paramedics, who report that the patient has partial-thickness burns on the chest and legs. The patient has also suffered smoke inhalation. What is the priority in the care of a patient who has been burned and suffered smoke inhalation? A) Pain B) Fluid balance C) Anxiety and fear D) Airway management

Ans: D Feedback: Systemic threats from a burn are the greatest threat to life. The ABCs of all trauma care apply during the early postburn period. While all options should be addressed, pain, fluid balance, and anxiety and fear do not take precedence over airway management.

A nurse is providing an educational class to a group of older adults at a community senior center. In an effort to prevent osteoporosis, the nurse should encourage participants to ensure that they consume the recommended adequate intake of what nutrients? Select all that apply. A) Vitamin B12 B) Potassium C) Calcitonin D) Calcium E) Vitamin D

Ans: D, E Feedback: A diet rich in calcium and vitamin D protects against skeletal demineralization. Intake of vitamin B12and potassium does not directly influence the risk for osteoporosis. Calcitonin is not considered to be a dietary nutrient

A bone biopsy has just been completed on a patient with suspected bone metastases. What assessment should the nurse prioritize in the immediate recovery period? A) Assessment for dehiscence at the biopsy site B) Assessment for pain C) Assessment for hematoma formation D) Assessment for infection

B

A nurse admits a patient who has a fracture of the nose that has resulted in a skin tear and involvement of the mucous membranes of the nasal passages. The orthopedic nurse is aware that this description likely indicates which type of fracture? A) Compression B) Compound C) Impacted D) Transverse

B

A nurses assessment of a patients knee reveals edema, tenderness, muscle spasms, and ecchymosis. The patient states that 2 days ago he ran 10 miles and now it really hurts to stand up. The nurse should plan care based on the belief that the patient has experienced what? A) A first-degree strain B) A second-degree strain C) A first-degree sprain D) A second-degree sprain

B

A patient has sustained a long bone fracture and the nurse is preparing the patients care plan. Which of the following should the nurse include in the care plan? A) Administer vitamin D and calcium supplements as ordered. B) Monitor temperature and pulses of the affected extremity. C) Perform passive range of motion exercises as tolerated. D) Administer corticosteroids as ordered.

B

A patient with a simple arm fracture is receiving discharge education from the nurse. What would the nurse instruct the patient to do? A) Elevate the affected extremity to shoulder level when at rest. B) Engage in exercises that strengthen the unaffected muscles. C) Apply topical anesthetics to accessible skin surfaces as needed. D) Avoid using analgesics so that further damage is not masked.

B

A public health nurse is organizing a campaign that will address the leading cause of musculoskeletal- related disability in the United States. The nurse should focus on what health problem? A) Osteoporosis B) Arthritis C) Hip fractures D) Lower back pain

B

An older adult patient has fallen in her home and is brought to the emergency department by ambulance with a suspected fractured hip. X-rays confirm a fracture of the left femoral neck. When planning assessments during the patients pre-surgical care, the nurse should be aware of the patients heightened risk of what complication? A) Osteomyelitis B) Avascular necrosis C) Phantom pain D) Septicemia

B

Which of the following is the most appropriate nursing intervention to facilitate healing in a patient who has suffered a hip fracture? A) Administer analgesics as required. B) Place a pillow between the patients legs when turning. C) Maintain prone positioning at all times. D) Encourage internal and external rotation of the affected leg.

B

A 42 year-old patient tells the nurse that she has found a painless lump in her right breast during her monthly self-examination. She says that she is afraid that she has cancer. Which assessment finding would most strongly suggest that this patient's lump is cancerous? A) Eversion of the right nipple and mobile mass B) A nonmobile mass with irregular edges C) A mobile mass that is soft and easily delineated D) Nonpalpable right axillary lymph nodes

B (Feedback: Breast cancer tumors are typically fixed, hard, and poorly delineated with irregular edges. A mobile mass that is soft and easily delineated is most commonly a fluid-filled benign cyst. Axillary lymph nodes may or may not be palpable on initial detection of a cancerous mass. Nipple retraction, not eversion, may be a sign of cancer.)

A patient at high risk for breast cancer is scheduled for an incisional biopsy in the outpatient surgery department. When the nurse is providing preoperative education, the patient asks why an incisional biopsy is being done instead of just removing the mass. What would be the nurse's best response? A) An incisional biopsy is performed because it's known to be less painful and more accurate than other forms of testing. B) An incisional biopsy is performed to confirm a diagnosis and so that special studies can be done that will help determine the best treatment. C) An incisional biopsy is performed to assess the potential for recovery from a mastectomy. D) An incisional biopsy is performed on patients who are younger than the age of 40 and who are otherwise healthy.

B (Feedback: Incisional biopsy surgically removes a portion of a mass. This is performed to confirm a diagnosis and to conduct special studies that will aid in determining treatment. Incisional biopsies cannot always remove the whole mass, nor is it always beneficial to the patient to do so. The procedure is not chosen because of the potential for pain, the possibility of recovery from mastectomy, or the patient's age.)

When caring for a patient who had a hemorrhagic stroke, close monitoring of vital signs and neurologic changes is imperative. What is the earliest sign of deterioration in a patient with a hemorrhagic stroke of which the nurse should be aware? A) Generalized pain B) Alteration in level of consciousness (LOC) C) Tonic-clonic seizures D) Shortness of breath

B (Feedback: Alteration in LOC is the earliest sign of deterioration in a patient after a hemorrhagic stroke, such as mild drowsiness, slight slurring of speech, and sluggish papillary reaction. Sudden headache may occur, but generalized pain is less common. Seizures and shortness of breath are not identified as early signs of hemorrhagic stroke.)

The nurse is caring for a patient recovering from an ischemic stroke. What intervention best addresses a potential complication after an ischemic stroke? A) Providing frequent small meals rather than three larger meals B) Teaching the patient to perform deep breathing and coughing exercises C) Keeping a urinary catheter in situ for the full duration of recovery D) Limiting intake of insoluble fiber

B (Feedback: Because pneumonia is a potential complication of stroke, deep breathing and coughing exercises should be encouraged unless contraindicated. No particular need exists to provide frequent meals and normally fiber intake should not be restricted. Urinary catheters should be discontinued as soon as possible.)

The nurse is planning the care of a patient with hyperthyroidism. What should the nurse specify in the patients meal plan? A) A clear liquid diet, high in nutrients B) Small, frequent meals, high in protein and calories C) Three large, bland meals a day D) A diet high in fiber and plant-sourced fat

B Feedback: A patient with hyperthyroidism has an increased appetite. (Hyper=increased metabolic rate) The patient should be counseled to consume several small, well-balanced meals. High-calorie, high-protein foods are encouraged. A clear liquid diet would not satisfy the patients caloric or hunger needs. A diet rich in fiber and fat should be avoided because these foods may lead to GI upset or increase peristalsis.

The nurse is caring for a patient with Addison's disease who is scheduled for discharge. When teaching the patient about hormone replacement therapy, the nurse should address what topic? A) The possibility of precipitous weight gain B) The need for lifelong steroid replacement C) The need to match the daily steroid dose to immediate symptoms D) The importance of monitoring liver function

B Feedback: Because of the need for lifelong replacement of adrenal cortex hormones to prevent addisonian crises, the patient and family members receive explicit education about the rationale for replacement therapy and proper dosage. Doses are not adjusted on a short-term basis. Weight gain and hepatotoxicity are not common adverse effects. Addison had to be on steroids for the rest of her life.

A man comes to the clinic complaining that he is having difficulty obtaining an erection. When reviewing the patient's history, what might the nurse note that contributes to erectile dysfunction? A) The patient has been treated for a UTI twice in the past year. B) The patient has a history of hypertension. C) The patient is 66 years old. D) The patient leads a sedentary lifestyle.

B Feedback: Past history of infection and lack of exercise do not contribute to impotence. With advancing age, sexual function and libido and potency decrease somewhat, but this is not the primary reason for impotence. Vascular problems cause about half the cases of impotence in men older than 50 years; hypertension is a major cause of such problems.

The physician has ordered a fluid deprivation test for a patient suspected of having diabetes insipidus. During the test, the nurse should prioritize what assessments? A) Temperature and oxygen saturation B) Heart rate and BP C) Breath sounds and bowel sounds D) Color, warmth, movement, and sensation of extremities

B Feedback: The fluid deprivation test is carried out by withholding fluids for 8 to 12 hours or until 3% to 5% of the body weight is lost. The patients condition needs to be monitored frequently during the test, and the test is terminated if tachycardia, excessive weight loss, or hypotension develops. Consequently, BP and heart rate monitoring are priorities over the other listed assessments.

A patient with a diagnosis of syndrome of inappropriate antidiuretic hormone secretion (SIADH) is being cared for on the critical care unit. The priority nursing diagnosis for a patient with this condition is what? A) Risk for peripheral neurovascular dysfunction B) Excess fluid volume C) Hypothermia D) Ineffective airway clearance

B Feedback: The priority nursing diagnosis for a patient with SIADH is excess fluid volume, as the patient retains fluids and develops a sodium deficiency. Restricting fluid intake is a typical intervention for managing this syndrome. Temperature imbalances are not associated with SIADH. The patient is not at risk for neurovascular dysfunction or a compromised airway.

A patient is brought to the ER following a motor vehicle accident in which he sustained head trauma. Preliminary assessment reveals a vision deficit in the patients left eye. The nurse should associate this abnormal finding with trauma to which of the following cerebral lobes? A) Temporal B) Occipital C) Parietal D) Frontal

B Hearing is the temporal

When assessing a patients peripheral nerve function, the nurse uses an instrument to prick the fat pad at the top of the patients small finger. This action will assess which of the following nerves? A) Radial B) Ulnar C) Median D) Tibial

B (Think of where pulses are located.)

The nursing educator is reviewing the signs and symptoms of heat stroke with a group of nurses who provide care in a desert region. The educator should describe what sign or symptom? A) Hypertension with a wide pulse pressure B) Anhidrosis C) Copious diuresis D) Cheyne-Stokes respirations

B Heat stroke is manifested by anhidrosis confusion, bizarre behavior, coma, elevated body temperature, hot dry skin, tachypnea, hypotension, and tachycardia. This health problem is not associated with anhidrosis or Cheyne-Stokes respirations.

A nurse is triaging patients after a chemical leak at a nearby fertilizer factory. The guiding principle of this activity is what? A) Assigning a high priority to the most critical injuries B) Doing the greatest good for the greatest number of people C) Allocating resources to the youngest and most critical D) Allocating resources on a first come, first served basis

B In non disaster situations, health care workers assign a high priority and allocate the most resources to those who are the most critically ill. However, in a disaster, when health care providers are faced with a large number of casualties, the fundamental principle guiding resource allocation is to do the greatest good for the greatest number of people. A first come, first served approach is unethical.

A nurse is caring for patients exposed to a terrorist attack involving chemicals. The nurse has been advised that personal protective equipment must be worn in order to give the highest level of respiratory protection with a lesser level of skin and eye protection. What level protection is this considered? A) Level A B) Level B C) Level C D) Level D

B Level B personal protective equipment provides the highest level of respiratory protection, with a lesser level of skin and eye protection. Level A provides the highest level of respiratory, mucous membrane, skin, and eye protection. Level C incorporates the use of an air-purified respirator, a chemical resistant coverall with splash hood, chemical resistant gloves, and boots. Level D is the same as a work uniform.

A patient is brought to the ED by ambulance with a gunshot wound to the abdomen. The nurse knows that the most common hollow organ injured in this type of injury is what? A) Liver B) Small bowel C) Stomach D) Large bowel

B Penetrating abdominal wounds have a high incidence of injury to hollow organs, especially the small bowel. The liver is also injured frequently, but it is a solid organ.

You are caring for a patient who has just been told that his illness is progressing and nothing more can be done for him. After the physician leaves, the patient asks you to stay with him for a while. The patient becomes tearful and tries several times to say something, but cannot get the words out. What would be an appropriate response for you to make at this time? A) "Can I give you some advice?" B) "Do you need more time to think about this?" C) "Is there anything you want to say?" D) "I have cared for lots of patients in your position. It will get easier."

B) "Do you need more time to think about this?" Prompt gently: "Do you need more time to think about this?" Giving advice is inappropriate and it is obvious from the scenario that the patient has something to say. Referring to other patients negates the patient's feelings at this time.

A patient with herpes simplex virus encephalitis (HSV) has been admitted to the ICU. What medication would the nurse expect the physician to order for the treatment of this disease process? A) Cyclosporine (Neoral) B) Acyclovir (Zovirax) C) Cyclobenzaprine (Flexeril) D) Ampicillin (Principen)

B) Acyclovir (Zovirax) Dim lighting and reduce environmental stimuli

A patient has suffered a muscle strain and is complaining of pain that she rates at 6 on a 10-point scale. The nurse should recommend what action? A) Taking an opioid analgesic as ordered B) Applying a cold pack to the injured site C) Performing passive ROM exercises D) Applying a heating pad to the affected muscle

B) Applying a cold pack to the injured site Most pain can be relieved by elevating the involved part, applying cold packs, and administering analgesics as prescribed. Heat may exacerbate the pain by increasing blood circulation, and ROM exercises would likely be painful. Analgesia is likely necessary, but NSAIDs would be more appropriate than opioids.

The orthopedic surgeon has prescribed balanced skeletal traction for a patient. What advantage is conferred by balanced traction? A) Balanced traction can be applied at night and removed during the day. B) Balanced traction allows for greater patient movement and independence than other forms of traction. C) Balanced traction is portable and may accompany the patients movements. D) Balanced traction facilitates bone remodeling in as little as 4 days.

B) Balanced traction allows for greater patient movement and independence than other forms of traction. Often, skeletal traction is balanced traction, which supports the affected extremity, allows for some patient movement, and facilitates patient independence and nursing care while maintaining effective traction. It is not portable, however, and it cannot be removed. Bone remodeling takes longer than 4 days.

A nurse is reviewing a patients activities of daily living prior to discharge from total hip replacement. The nurse should identify what activity as posing a potential risk for hip dislocation? A) Straining during a bowel movement B) Bending down to put on socks C) Lifting items above shoulder level D) Transferring from a sitting to standing position

B) Bending down to put on socks Bending to put on socks or shoes can cause hip dislocation. None of the other listed actions poses a serious threat to the integrity of the new hip.

A nurse is caring for a patient who has been diagnosed with leukemia. The nurse's most recent assessment reveals the presence of ecchymosis on the patient's sacral area and petechiae in her forearms. In addition to informing the patient's primary care provider, the nurse should perform what action? A) Initiate measures to prevent venous thromboembolism (VTE). B) Check the patient's most recent platelet level. C) Place the patient on protective isolation. D) Ambulate the patient to promote circulatory function.

B) Check the patient's most recent platelet level. The patient's signs are suggestive of thrombocytopenia, thus the nurse should check the patient's most recent platelet level. VTE is not a risk and this does not constitute a need for isolation. Ambulation and activity may be contraindicated due to the risk of bleeding.

A nurse is performing an admission assessment on a patient with stage 3 HIV. After assessing the patients gastrointestinal system and analyzing the data, what is most likely to be the priority nursing diagnosis? A) Acute Abdominal Pain B) Diarrhea C) Bowel Incontinence D) Constipation

B) Diarrhea

The nurses assessment of a patient with thyroidectomy suggests tetany and a review of the most recent blood work corroborate this finding. The nurse should prepare to administer what intervention? A) Oral calcium chloride and vitamin D B) IV calcium gluconate C) STAT levothyroxine D) Administration of parathyroid hormone (PTH)

B) IV calcium gluconate

An adult patient has presented to the health clinic with a complaint of a firm, painless cervical lymph node. The patient denies any recent infectious diseases. What is the nurse's most appropriate response to the patient's complaint? A) Call 911. B) Promptly refer the patient for medical assessment. C) Facilitate a radiograph of the patient's neck and have the results forwarded to the patient's primary care provider. D) Encourage the patient to track the size of the lymph node and seek care in 1 week.

B) Promptly refer the patient for medical assessment Hodgkin lymphoma usually begins as an enlargement of one or more lymph nodes on one side of the neck. The individual nodes are painless and firm but not hard. Prompt medical assessment is necessary if a patient has this presentation. However, there is no acute need to call 911. Delaying care for 1 week could have serious consequences and x-rays are not among the common diagnostic tests.

As a member of the stroke team, the nurse knows that thrombolytic therapy carries the potential for benefit and for harm. The nurse should be cognizant of what contraindications for thrombolytic therapy? Select all that apply. A) INR above 1.0 B) Recent intracranial pathology C) Sudden symptom onset D) Current anticoagulation therapy E) Symptom onset greater than 3 hours prior to admission

B) Recent intracranial pathology D) Current anticoagulation therapy E) Symptom onset greater than 3 hours prior to admission

An adult patient's abnormal complete blood count (CBC) and physical assessment have prompted the primary care provider to order a diagnostic workup for Hodgkin lymphoma. The presence of what assessment finding is considered diagnostic of the disease? A) Schwann cells B) Reed-Sternberg cells C) Lewy bodies D) Loops of Henle

B) Reed-Sternberg cells The malignant cell of Hodgkin lymphoma is the Reed-Sternberg cell, a gigantic tumor cell that is morphologically unique and thought to be of immature lymphoid origin. It is the pathologic hallmark and essential diagnostic criterion. Schwann cells exist in the peripheral nervous system and Lewy bodies are markers of Parkinson disease. Loops of Henle exist in nephrons.

A patient with HIV is admitted to the hospital because of chronic severe diarrhea. The nurse caring for this patient should expect the physician to order what drug for the management of the patients diarrhea? A) Zithromax B) Sandostatin C) Levaquin D) Biaxin

B) Sandostatin

A nurse is caring for a patient receiving skeletal traction. Due to the patients severe limits on mobility, the nurse has identified a risk for atelectasis or pneumonia. What intervention should the nurse provide in order to prevent these complications? A) Perform chest physiotherapy once per shift and as needed. B) Teach the patient to perform deep breathing and coughing exercises. C) Administer prophylactic antibiotics as ordered. D) Administer nebulized bronchodilators and corticosteroids as ordered.

B) Teach the patient to perform deep breathing and coughing exercises. To prevent these complications, the nurse should educate the patient about performing deep-breathing and coughing exercises to aid in fully expanding the lungs and clearing pulmonary secretions. Antibiotics, bronchodilators, and steroids are not used on a preventative basis and chest physiotherapy is unnecessary and implausible for a patient in traction.

A patient with metastatic cancer has developed trigeminal neuralgia and is taking carbamazepine (Tegretol) for pain relief. What principle applies to the administration of this medication? A) Tegretol is not known to have serious adverse effects. B) The patient should be monitored for bone marrow depression. C) Side effects of the medication include renal dysfunction. D) The medication should be first taken in the maximum dosage form to be effective.

B) The patient should be monitored for bone marrow depression. Anticonvulsant meds for neurological pain FIFTH cranial nerve Monitor blood levels of meds

A nurse has created a plan of care for an immunodeficient patient, specifying that care providers take the patient's pulse and respiratory rate for a full minute. What is the rationale for this aspect of care? A) Respirations affect heart rate in immunodeficient patients. B) These patients' blunted inflammatory responses can cause subtle changes in status. C) Hemodynamic instability is one of the main complications of immunodeficiency. D) Immunodeficient patients are prone to ventricular tachycardia and atrial fibrillation.

B) These patients' blunted inflammatory responses can cause subtle changes in status. Pulse rate and respiratory rate should be counted for a full minute, because subtle changes can signal deterioration in the patient's clinical status. The rationale for this action is not because of the relationship between heart rate and respirations. These patients do not have a greatly increased risk of hemodynamic instability or dysrhythmias.)

A patient has just died following urosepsis that progressed to septic shock. The patient's spouse says, "I knew this was coming, but I feel so numb and hollow inside." The nurse should know that these statements are characteristic of what? A) Complicated grief and mourning B) Uncomplicated grief and mourning C) Depression stage of dying D) Acceptance stage of dying

B) Uncomplicated grief and mourning Uncomplicated grief and mourning are characterized by emotional feelings of sadness, anger, guilt, and numbness; physical sensations, such as hollowness in the stomach and tightness in the chest, weakness, and lack of energy; cognitions that include preoccupation with the loss and a sense of the deceased as still present; and behaviors such as crying, visiting places that are reminders of the deceased, social withdrawal, and restless overactivity. Complicated grief and mourning occur at a prolonged time after the death. The spouse's statement does not clearly suggest depression or acceptance.

A nurse who is a member of the local disaster response team is learning about blast injuries. The nurse should plan for what event that occurs in the tertiary phase of the blast injury? A) Victims pre-existing medical conditions are exacerbated. B) Victims are thrown by the pressure wave. C) Victims experience burns from the blast. D) Victims suffer injuries caused by debris or shrapnel from the blast.

B) Victims are thrown by the pressure wave.

A nurse in a busy emergency department provides care for many patients who present with contusions, strains, or sprains. Treatment modalities that are common to all of these musculoskeletal injuries include which of the following? Select all that apply. A) Massage B) Applying ice C) Compression dressings D) Resting the affected extremity E) Corticosteroids F) Elevating the injured limb

B, C, D, F

A patient with ovarian cancer is admitted to the hospital for surgery and the nurse is completing the patients health history. What clinical manifestation would the nurse expect to assess? A) Fish-like vaginal odor B) Increased abdominal girth C) Fever and chills D) Lower abdominal pelvic pain

B. Clinical manifestations of ovarian cancer include enlargement of the abdomen from an accumulation of fluid. Flatulence and feeling full after a light meal are significant symptoms. In bacterial vaginosis, a fish-like odor, which is noticeable after sexual intercourse or during menstruation, occurs as a result of a rise in the vaginal pH. Fever, chills, and abdominal pelvic pain are atypical.

A middle-aged female patient has been offered testing for HIV/AIDS upon admission to the hospital for an unrelated health problem. The nurse observes that the patient is visibly surprised and embarrassed by this offer. How should the nurse best respond? A) Most women with HIV don't know they have the disease. If you have it, it's important we catch it early. B) This testing is offered to every adolescent and adult regardless of their lifestyle, appearance or history. C) The rationale for this testing is so that you can begin treatment as soon as testing comes back, if its positive. D) You're being offered this testing because you are actually in the prime demographic for HIV infection.

B. Feedback: Because patients may be reluctant to discuss risk-taking behavior, routine screening should be offered to all women between the ages of 13 to 64 years in all health care settings. Assuring a woman that the offer of testing is not related to a heightened risk may alleviate her anxiety. Middle-aged women are not the prime demographic for HIV infection. The nurse should avoid causing fear by immediately discussing treatment or the fact that many patients are unaware of their diagnosis.

When teaching patients about the risk factors of cervical cancer, what would the nurse identify as the most important risk factor? A) Late childbearing B) Human papillomavirus (HPV) C) Postmenopausal bleeding D) Tobacco use

B. Feedback: HPV is the most salient risk factor for cervical cancer, exceeding the risks posed by smoking, late childbearing, and postmenopausal bleeding.

Osteoarthritis:

Begins third decade of life and peaks the 5th/6th

A nurse is performing a shift assessment on an elderly patient who is recovering after surgery for a hip fracture. The nurse notes that the patient is complaining of chest pain, has an increased heart rate, and increased respiratory rate. The nurse further notes that the patient is febrile and hypoxic, coughing, and producing large amounts of thick, white sputum. The nurse recognizes that this is a medical emergency and calls for assistance, recognizing that this patient is likely demonstrating symptoms of what complication? A) Avascular necrosis of bone B) Compartment syndrome C) Fat embolism syndrome D) Complex regional pain syndrome

C

A patient has been experiencing progressive increases in knee pain and diagnostic imaging reveals a worsening effusion in the synovial capsule. The nurse should anticipate which of the following? A) Arthrography B) Knee biopsy C) Arthrocentesis D) Electromyography

C

A patient is being treated for a fractured hip and the nurse is aware of the need to implement interventions to prevent muscle wasting and other complications of immobility. What intervention best addresses the patients need for exercise? A) Performing gentle leg lifts with both legs B) Performing massage to stimulate circulation C) Encouraging frequent use of the overbed trapeze D) Encouraging the patient to log roll side to side once per hour

C

A school nurse is assessing a student who was kicked in the shin during a soccer game. The area of the injury has become swollen and discolored. The triage nurse recognizes that the patient has likely sustained what? A) Sprain B) Strain C) Contusion D) Dislocation

C

Six weeks after an above-the-knee amputation (AKA), a patient returns to the outpatient office for a routine postoperative checkup. During the nurses assessment, the patient reports symptoms of phantom pain. What should the nurse tell the patient to do to reduce the discomfort of the phantom pain? A) Apply intermittent hot compresses to the area of the amputation. B) Avoid activity until the pain subsides. C) Take opioid analgesics as ordered. D) Elevate the level of the amputation site.

C

The human body is designed to protect its vital parts. A fracture of what type of bone may interfere with the protection of vital organs? A) Long bones B) Short bones C) Flat bones D) Irregular bones

C

The results of a nurses musculoskeletal examination show an increase in the lumbar curvature of the spine. The nurse should recognize the presence of what health problem? A) Osteoporosis B) Kyphosis C) Lordosis D) Scoliosis

C

A 23-year-old woman comes to the free clinic stating I think I have a lump in my breast. Do I have cancer? The nurse instructs the patient that a diagnosis of breast cancer is confirmed by what? A) Supervised breast self-examination B) Mammography C) Fine-needle aspiration D) Chest x-ray

C (Feedback: Fine-needle aspiration and biopsy provide cells for histologic examination to confirm a diagnosis, although falsenegative and falsepositive findings are possibilities. A breast self-examination, if done regularly, is the most reliable method for detecting breast lumps early, but is not diagnostic of cancer. Mammography is used to detect tumors that are too small to palpate. Chest x-rays can be used to pinpoint rib metastasis. Neither test is considered diagnostic of breast cancer, however.)

During a recent visit to the clinic a woman presents with erythema of the nipple and areola on the right breast. She states this started several weeks ago and she was fearful of what would be found. The nurse should promptly refer the patient to her primary care provider because the patient's signs and symptoms are suggestive of what health problem? A) Peau d'orange B) Nipple inversion C) Paget's disease D) Acute mastitis

C (Feedback: Paget's disease presents with erythema of the nipple and areola. Peau d'orange, which is associated with breast cancer, is caused by interference with lymphatic drainage, but does not cause these specific signs. Nipple inversion is considered normal if long-standing; if it is associated with fibrosis and is a recent development, malignancy is suspected. Acute mastitis is associated with lactation, but it may occur at any age.)

The nurse is assessing a patient diagnosed with Graves disease. What physical characteristics of Graves disease would the nurse expect to find? A) Hair loss B) Moon face C) Bulging eyes D) Fatigue

C Feedback: Clinical manifestations of the endocrine disorder Graves disease include exophthalmos (bulging eyes) and fine tremor in the hands. Graves disease is not associated with hair loss, a moon face, or fatigue.

The nurse is caring for a patient with a diagnosis of Addison's disease. What sign or symptom is most closely associated with this health problem? A) Truncal obesity B) Hypertension C) Muscle weakness D) Moon face

C Feedback: Patients with Addison's disease demonstrate muscular weakness, anorexia, gastrointestinal symptoms, fatigue, emaciation, dark pigmentation of the skin, and hypotension. Patients with Cushing syndrome demonstrate truncal obesity, moon face, acne, abdominal striae, and hypertension. Addison was bronze in appearance and was very tired, weak and thin.

A nurse caring for a patient with diabetes insipidus is reviewing laboratory results. What is an expected urinalysis finding? A) Glucose in the urine B) Albumin in the urine C) Highly dilute urine D) Leukocytes in the urine

C Feedback: Patients with diabetes insipidus produce an enormous daily output of very dilute, water-like urine with a specific gravity of 1.001 to 1.005. The urine contains no abnormal substances such as glucose or albumin. Leukocytes in the urine are not related to the condition of diabetes insipidus, but would indicate a urinary tract infection, if present in the urine.

A nurse practitioner is assessing a 55-year-old male patient who is complaining of perineal discomfort, burning, urgency, and frequency with urination. The patient states that he has pain with ejaculation. The nurse knows that the patient is exhibiting symptoms of what? A) Varicocele B) Epididymitis C) Prostatitis D) Hydrocele

C Feedback: Perineal discomfort, burning, urgency, frequency with urination, and pain with ejaculation is indicative of prostatitis. A varicocele is an abnormal dilation of the pampiniform venous plexus and the internal spermatic vein in the scrotum (the network of veins from the testis and the epididymis that constitute part of the spermatic cord). Epididymitis is an infection of the epididymis that usually descends from an infected prostate or urinary tract; it also may develop as a complication of gonorrhea. A hydrocele is a collection of fluid, generally in the tunica vaginalis of the testis, although it also may collect within the spermatic cord.

A nurse is assessing a patient who presented to the ED with priapism. The student nurse is aware that this condition is classified as a urologic emergency because of the potential for what? A) Urinary tract infection B) Chronic pain C) Permanent vascular damage D) Future erectile dysfunction

C Feedback: The ischemic form of priapism, which is described as nonsexual, persistent erection with little or no cavernous blood flow, must be treated promptly to prevent permanent damage to the penis. Priapism has not been indicated in the development of UTIs, chronic pain, or erectile dysfunction.

A patient is prescribed corticosteroid therapy. What would be priority information for the nurse to give the patient who is prescribed long-term corticosteroid therapy? A) The patients diet should be low protein with ample fat. B) The patient may experience short-term changes in cognition. C) The patient is at an increased risk for developing infection. D) The patient is at a decreased risk for development of thrombophlebitis and thromboembolism.

C Feedback: The patient is at increased risk of infection and masking of signs of infection. The cardiovascular effects of corticosteroid therapy may result in development of thrombophlebitis or thromboembolism. Diet should be high in protein with limited fat. Changes in appearance usually disappear when therapy is no longer necessary. Cognitive changes are not common adverse effects.

A group of medical nurses are being certified in their response to potential bioterrorism. The nurses learn that if a patient is exposed to the smallpox virus he or she becomes contagious at what time? A) 6 to 12 hours after exposure B) When pustules form C) After a rash appears D) When the patient becomes febrile

C A patient is contagious after a rash develops, which initially develops on the face, mouth, pharynx, and forearms. The patient exposed to the smallpox virus is not contagious immediately after exposure; only when pustules form, or with a body temperature of 38C.

A nurse has had contact with a patient who developed smallpox and became febrile after a terrorist attack. This nurse will require what treatment? A) Watchful waiting B) Treatment with colony-stimulating factors (CSFs) C) Vaccination D) Treatment with ceftriaxone

C All people who have had household or face-to-face contact with a patient with small pox after the fever begins should be vaccinated within 4 days to prevent infection and death. Watchful waiting would be inappropriate and CSFs are not used for treatment. Vaccination, rather than antibiotics, is the treatment of choice.

A patient with a fractured femur presenting to the ED exhibits cool, moist skin, increased heart rate, and falling BP. The care team should consider the possibility of what complication of the patients injuries? A) Myocardial infarction B) Hypoglycemia C) Hemorrhage D) Peritonitis

C The signs and symptoms the patient is experiencing suggest a volume deficit from an internal bleed. That the symptoms follow an acute injury suggests hemorrhage rather than myocardial infarction or hypoglycemia. Peritonitis would be an unlikely result of a femoral fracture.

A patient has just been told that her illness is terminal. The patient tearfully states, "I can't believe I am going to die. Why me?" What is your best response? A) "I know how you are feeling." B) "You have lived a long life." C) "This must be very difficult for you." D) "Life can be so unfair."

C) "This must be very difficult for you." The most important intervention the nurse can provide is listening empathetically. To communicate effectively, the nurse should ask open-ended questions and acknowledge the patient's fears. Deflecting the statement or providing false sympathy must be avoided.

The clinical nurse educator is presenting health promotion education to a patient who will be treated for non-Hodgkin lymphoma on an outpatient basis. The nurse should recommend which of the following actions? A) Avoiding direct sun exposure in excess of 15 minutes daily B) Avoiding grapefruit juice and fresh grapefruit C) Avoiding highly crowded public places D) Using an electric shaver rather than a razor

C) Avoiding highly crowded public places The risk of infection is significant for these patients, not only from treatment-related myelosuppression but also from the defective immune response that results from the disease itself. Limiting infection exposure is thus necessary. The need to avoid grapefruit is dependent on the patient's medication regimen. Sun exposure and the use of razors are not necessarily contraindicated.

A patient is admitted to the unit in traction for a fractured proximal femur and requires traction prior to surgery. What is the most appropriate type of traction to apply to a fractured proximal femur? A) Russell's traction B) Dunlop's traction C) Bucks extension traction D) Cervical head halter

C) Bucks extension traction Bucks extension is used for fractures of the proximal femur. Russells traction is used for lower leg fractures. Dunlops traction is applied to the upper extremity for supracondylar fractures of the elbow and humerus. Cervical head halters are used to stabilize the neck.

A patient is complaining of pain in her casted leg. The nurse has administered analgesics and elevated the limb. Thirty minutes after administering the analgesics, the patient states the pain is unrelieved. The nurse should identify the warning signs of what complication? A) Subcutaneous emphysema B) Skin breakdown C) Compartment syndrome D) Disuse syndrome

C) Compartment syndrome Compartment syndrome may manifest as unrelenting, uncontrollable pain. This presentation of pain is not suggestive of disuse syndrome or skin breakdown. Subcutaneous emphysema is not a complication of casting.

A nurse is caring for a patient who has a leg cast. The nurse observes that the patient uses a pencil to scratch the skin under the edge of the cast. How should the nurse respond to this observation? A) Allow the patient to continue to scratch inside the cast with a pencil but encourage him to be cautious. B) Give the patient a sterile tongue depressor to use for scratching instead of the pencil. C) Encourage the patient to avoid scratching, and obtain an order for an antihistamine if severe itching persists. D) Obtain an order for a sedative, such as lorazepam (Ativan), to prevent the patient from scratching.

C) Encourage the patient to avoid scratching, and obtain an order for an antihistamine if severe itching persists. Scratching should be discouraged because of the risk for skin breakdown or damage to the cast. Most patients can be discouraged from scratching if given a mild antihistamine, such as diphenhydramine, to relieve itching. Benzodiazepines would not be given for this purpose.

A nurse at a long-term care facility is amending the care plan of a resident who has just been diagnosed with essential thrombocythemia (ET). The nurse should anticipate the administration of what medication? A) Dalteparin B) Allopurinol C) Hydroxyurea D) Hydrochlorothiazide

C) Hydroxyurea Hydroxyurea is effective in lowering the platelet count for patients with ET. Dalteparin, allopurinol, and HCTZ do not have this therapeutic effect.

An emergency department nurse is triaging a 77-year-old man who presents with uncharacteristic fatigue as well as back and rib pain. The patient denies any recent injuries. The nurse should recognize the need for this patient to be assessed for what health problem? A) Hodgkin disease B) Non-Hodgkin lymphoma C) Multiple myeloma D) Acute thrombocythemia

C) Multiple myeloma Back pain, which is often a presenting symptom in multiple myeloma, should be closely investigated in older patients. The lymphomas and bleeding disorders do not typically present with the primary symptom of back pain or rib pain.

The nurse is caring for a patient with a diagnosis of Addison's disease. What sign or symptom is most closely associated with this health problem? A) Truncal obesity B) Hypertension C) Muscle weakness D) Moon face

C) Muscle weakness

The nurse is caring for a patient who underwent a total hip replacement yesterday. What should the nurse do to prevent dislocation of the new prosthesis? A) Keep the affected leg in a position of adduction. B) Have the patient reposition himself independently. C) Protect the affected leg from internal rotation. D) Keep the hip flexed by placing pillows under the patients knee.

C) Protect the affected leg from internal rotation. Abduction of the hip helps to prevent dislocation of a new hip joint. Rotation and adduction should be avoided. While the hip may be flexed slightly, it shouldnt exceed 90 degrees and maintenance of flexion isnt necessary. The patient may not be capable of safe independent repositioning at this early stage of recovery.

A patient has been admitted to the post-surgical unit following a thyroidectomy. To promote comfort and safety, how should the nurse best position the patient? A) Side-lying (lateral) with one pillow under the head B) Head of the bed elevated 30 degrees and no pillows placed under the head C) Semi-Fowlers with the head supported on two pillows D) Supine, with a small roll supporting the neck

C) Semi-Fowlers with the head supported on two pillows

A patient has a glomerular filtration rate (GFR) of 43 mL/min/1.73 m2 Based on this GFR, the nurse interprets that the patients chronic kidney disease is at what stage? A) Stage 1 B) Stage 2 C) Stage 3 D) Stage 4

C) Stage 3 Stages of chronic renal failure are based on the GFR. Stage 3 is defined by a GFR in the range of 30 to 59 mL/min/1.73 m2 This is considered a moderate decrease in GFR.

A 45-year-old woman has just undergone a radical hysterectomy for invasive cervical cancer. Prior to the surgery the physician explained to the patient that after the surgery a source of radiation would be placed near the tumor site to aid in reducing recurrence. What is the placement of the source of radiation called? A) Internal beam radiation B) Trachelectomy C) Brachytherapy D) External radiation

C. Feedback: Radiation, which is often part of the treatment to reduce recurrent disease, may be delivered by an external beam or by brachytherapy (method by which the radiation source is placed near the tumor) or both.

CA-125 detects ovarian cancer and many other types of cancers HER-2- Breast Cancer

CA 19-9 detects pancreas and GB cancer PSA- Prostate Cancer AFP- primary and metastatic liver cancer

A nurse is caring for a patient diagnosed with Meniere's disease. While completing a neurologic examination on the patient, the nurse assesses cranial nerve VIII. The nurse would be correct in identifying the function of this nerve as what? A) Movement of the tongue B) Visual acuity C) Sense of smell D) Hearing and equilibrium

D

A nurse is preparing to discharge a patient from the emergency department after receiving treatment for an ankle sprain. While providing discharge education, the nurse should encourage which of the following? A) Apply heat for the first 24 to 48 hours after the injury. B) Maintain the ankle in a dependent position. C) Exercise hourly by performing rotation exercises of the ankle. D) Keep an elastic compression bandage on the ankle.

D

A patient who has had an amputation is being cared for by a multidisciplinary rehabilitation team. What is the primary goal of this multidisciplinary team? A) Maximize the efficiency of care B) Ensure that the patients health care is holistic C) Facilitate the patients adjustment to a new body image D) Promote the patients highest possible level of function

D

A patient has presented for her annual mammogram. The patient voices concerns related to exposure to radiation. What should the nurse teach the patient about a mammogram? A) It does not use radiation. B) Radiation levels are safe as long as mammograms are performed only once per year. C) The negative effects of radiation do not accumulate until late in life. D) Radiation from a mammogram is equivalent to an hour of sunlight.

D (Feedback: The radiation exposure of mammogram is equivalent to about 1 hour of exposure to sunlight. Consequently, the benefits of mammography far outweigh any risks associated with the procedure. Negative consequences are insignificant, and do not accumulate later in life.)

Phases of Burn Injury

Emergent/resuscitative phase: time of injury to fluids (48 hours post is most critical) Acute/intermediate phase: diuresis start to wound closure Rehabilitation phase: return to optimal functioning

A patient has had a cast placed for the treatment of a humeral fracture. The nurses most recent assessment shows signs and symptoms of compartment syndrome. What is the nurses most appropriate action? A) Arrange for a STAT assessment of the patients serum calcium levels. B) Perform active range of motion exercises. C) Assess the patients joint function symmetrically. D) Contact the primary care provider immediately.

D Emergency situation.

A patient has been taking prednisone for several weeks after experiencing a hypersensitivity reaction. To prevent adrenal insufficiency, the nurse should ensure that the patient knows to do which of the following? A) Take the drug concurrent with levothyroxine (Synthroid). B) Take each dose of prednisone with a dose of calcium chloride. C) Gradually replace the prednisone with an OTC alternative. D) Slowly taper down the dose of prednisone, as ordered.

D Feedback: Corticosteroid dosages are reduced gradually (tapered) to allow normal adrenal function to return and to prevent steroid-induced adrenal insufficiency. There are no OTC substitutes for prednisone and neither calcium chloride nor levothyroxine addresses the risk of adrenal insufficiency.

A patient is 24 hours postoperative following prostatectomy and the urologist has ordered continuous bladder irrigation. What color of output should the nurse expect to find in the drainage bag? A) Red wine colored B) Tea colored C) Amber D) Light pink

D Feedback: The urine drainage following prostatectomy usually begins as a reddish pink, then clears to a light pink 24 hours after surgery.

The nurse is conducting a focused neurological assessment. When assessing the patients cranial nerve function, the nurse would include which of the following assessments? A) Assessment of hand grip B) Assessment of orientation to person, time, and place C) Assessment of arm drift D) Assessment of gag reflex

D XI and X will have difficulty swallowing and absent gag reflex

A patient is admitted to the ED who has been exposed to a nerve agent. The nurse should anticipate the STAT administration of what drug? A) Amyl nitrate B) Dimercaprol C) Erythromycin D) Atropine

D Atropine is administered when a patient is exposed to a nerve agent. Exposure to blood agents, such as cyanide, requires treatment with amyl nitrate, sodium nitrite, and sodium thiosulfate. Dimercaprol is administered IV for systemic toxicity and topically for skin lesions when exposed to vesicants. Erythromycin is an antibiotic, which is ineffective against nerve agents.

When assessing patients who are victims of a chemical agent attack, the nurse is aware that assessment findings vary based on the type of chemical agent. The chemical sulfur mustard is an example of what type of chemical warfare agent? A) Nerve agent B) Blood agent C) Pulmonary agent D) Vesicant

D Sulfur mustard is a vesicant chemical that causes blistering and results in burning, conjunctivitis, bronchitis, pneumonia, hematopoietic suppression, and death. Nerve agents include sarin, soman, tabun, VX, and organophosphates (pesticides). Hydrogen cyanide is a blood agent that has a direct effect on cellular metabolism, resulting in asphyxiation through alterations in hemoglobin. Chlorine is a pulmonary agent, which destroys the pulmonary membrane that separates the alveolus from the capillary bed.

A patient who has been exposed to anthrax is being treated in the local hospital. The nurse should prioritize what health assessments? A) Integumentary assessment B) Assessment for signs of hemorrhage C) Neurologic assessment D) Assessment of respiratory status

D The second stage of anthrax infection by inhalation includes severe respiratory distress, including stridor, cyanosis, hypoxia, diaphoresis, hypotension, and shock. The first stage includes flu-like symptoms. The second stage of infection by inhalation does not include headache, vomiting, or syncope. Inhalation by Anthrax is the worst exposure.

The nurse has been notified that the ED is expecting terrorist attack victims and that level D personal protective equipment is appropriate. What does level D PPE include? A) A chemical-resistant coverall with splash hood, chemical-resistant gloves, and boots B) A self-contained breathing apparatus (SCBA) and a fully encapsulating, vapor-tight, chemical resistant suit with chemical-resistant gloves and boots. C) The SCBA and a chemical-resistant suit, but the suit is not vapor tight D) The nurses typical work uniform

D The typical work uniform is appropriate for Level D protection

What is the purpose of burr holes in neurosurgical procedures? a. Make a bone flap in the skull b. Aspirate a brain abscess c. Evacuate a hematoma d. All of the above

D > In powerpoint

A 35-year-old male is admitted to the hospital complaining of severe headaches, vomiting, and testicular pain. His blood work shows reduced numbers of platelets, leukocytes, and erythrocytes, with a high proportion of immature cells. The nurse caring for this patient suspects a diagnosis of what? A) AML B) CML C) MDS D) ALL

D) ALL In acute lymphocytic leukemia (ALL), manifestations of leukemic cell infiltration into other organs are more common than with other forms of leukemia, and include pain from an enlarged liver or spleen, as well as bone pain. The central nervous system is frequently a site for leukemic cells; thus, patients may exhibit headache and vomiting because of meningeal involvement. Other extranodal sites include the testes and breasts. This particular presentation is not closely associated with acute myeloid leukemia (AML), chronic myeloid leukemia (CML), or myelodysplastic syndromes (MDS)

A nurse who works in the specialty of palliative care frequently encounters issues and situations that constitute ethical dilemmas. What issue has most often presented challenging ethical issues, especially in the context of palliative care? A) The increase in cultural diversity in the United States B) Staffing shortages in health care and questions concerning quality of care C) Increased costs of health care coupled with inequalities in access D) Ability of technology to prolong life beyond meaningful quality of life

D) Ability of technology to prolong life beyond meaningful quality of life The application of technology to prolong life has raised several ethical issues. The major question is, "Because we can prolong life through increasingly sophisticated technology, does it necessarily follow that we must do so?" The increase in cultural diversity has not raised ethical issues in health care. Similarly, costs and staffing issues are relevant, but not central to the most common ethical issues surrounding palliative care.

A 66-year-old patient is in a hospice receiving palliative care for lung cancer which has metastasized to the patient's liver and bones. For the past several hours, the patient has been experiencing dyspnea. What nursing action is most appropriate to help to relive the dyspnea the patient is experiencing? A) Administer a bolus of normal saline, as ordered. B) Initiate high-flow oxygen therapy. C) Administer high doses of opioids. D) Administer bronchodilators and corticosteroids, as ordered.

D) Administer bronchodilators and corticosteroids, as ordered. Bronchodilators and corticosteroids help to improve lung function as well as low doses of opioids. Low-flow oxygen often provides psychological comfort to the patient and family. A fluid bolus is unlikely to be of benefit.

A nurse is caring for a patient who is in skeletal traction. To prevent the complication of skin breakdown in a patient with skeletal traction, what action should be included in the plan of care? A) Apply occlusive dressings to the pin sites. B) Encourage the patient to push up with the elbows when repositioning. C) Encourage the patient to perform isometric exercises once a shift. D) Assess the pin insertion site every 8 hours.

D) Assess the pin insertion site every 8 hours. The pin insertion site should be assessed every 8 hours for inflammation and infection. Loose cover dressings should be applied to pin sites. The patient should be encouraged to use the overhead trapeze to shift weight for repositioning. Isometric exercises should be done 10 times an hour while awake.

A clinic patient is being treated for polycythemia vera and the nurse is providing health education. What practice should the nurse recommend in order to prevent the complications of this health problem? A) Avoiding natural sources of vitamin K B) Avoiding altitudes of ³1500 feet (457 meters) C) Performing active range of motion exercises daily D) Avoiding tight and restrictive clothing on the legs

D) Avoiding tight and restrictive clothing on the legs Because of the risk of DVT, patients with polycythemia vera should avoid tight and restrictive clothing. There is no need to avoid foods with vitamin K or to avoid higher altitudes. Activity levels should be maintained, but there is no specific need for ROM exercises.

A clinic nurse is working with a patient who has a long-standing diagnosis of polycythemia vera. How can the nurse best gauge the course of the patient's disease? A) Document the color of the patient's palms and face during each visit. B) Follow the patient's erythrocyte sedimentation rate over time. C) Document the patient's response to erythropoietin injections. D) Follow the trends of the patient's hematocrit.

D) Follow the trends of the patient's hematocrit . The course of polycythemia vera can be best ascertained by monitoring the patient's hematocrit, which should remain below 45%. Erythropoietin injections would exacerbate the condition. Skin tone should be observed, but is a subjective assessment finding. The patient's ESR is not relevant to the course of the disease.

A nurse is caring for a patient who has had a total hip replacement. The nurse is reviewing health education prior to discharge. Which of the patients statements would indicate to the nurse that the patient requires further teaching? A) I will need to keep several pillows between my legs at night. B) I need to remember not to cross my legs. Its such a habit. C) The occupational therapist is showing me how to use a sock puller to help me get dressed. D) I will need my husband to assist me in getting off the low toilet seat at home.

D) I will need my husband to assist me in getting off the low toilet seat at home. I need to remember not to cross my legs. Its such a habit.The occupational therapist is showing me how to use a sock puller to help me get dressed. I will need my husband to assist me in getting off the low toilet seat at home.D Feedback: To prevent hip dislocation after a total hip replacement, the patient must avoid bending the hips beyond 90 degrees. Assistive devices, such as a raised toilet seat, should be used to prevent severe hip flexion. Using an abduction pillow or placing several pillows between the legs reduces the risk of hip dislocation by preventing adduction and internal rotation of the legs. Likewise, teaching the patient to avoid crossing the legs also reduces the risk of hip dislocation. A sock puller helps a patient get dressed without flexing the hips beyond 90 degrees.

A nurse is caring for a patient who is recovering in the hospital following orthopedic surgery. The nurse is performing frequent assessments for signs and symptoms of infection in the knowledge that the patient faces a high risk of what infectious complication? A) Cellulitis B) Septic arthritis C) Sepsis D) Osteomyelitis

D) Osteomyelitis Infection is a risk after any surgery, but it is of particular concern for the postoperative orthopedic patient because of the risk of osteomyelitis. Orthopedic patients do not have an exaggerated risk of cellulitis, sepsis, or septic arthritis when compared to other surgical patients.

A nurse is assessing the neurovascular status of a patient who has had a leg cast recently applied. The nurse is unable to palpate the patients dorsalis pedis or posterior tibial pulse and the patients foot is pale. What is the nurses most appropriate action? A) Warm the patients foot and determine whether circulation improves. B) Reposition the patient with the affected foot dependent. C) Reassess the patients neurovascular status in 15 minutes. D) Promptly inform the primary care provider.

D) Promptly inform the primary care provider. Signs of neurovascular dysfunction warrant immediate medical follow-up. It would be unsafe to delay. Warming the foot or repositioning the patient may be of some benefit, but the care provider should be informed first.

A 15-year-old is admitted to the renal unit with a diagnosis of post infectious glomerular disease. The nurse should recognize that this form of kidney disease may have been precipitated by what event? A) Psychosocial stress B) Hypersensitivity to an immunization C) Menarche D) Streptococcal infection

D) Streptococcal infection

The nurse is providing preoperative education for a patient diagnosed with endometriosis. A hysterectomy has been scheduled. What education topic should the nurse be sure to include for this patient? A) Menstrual periods will continue to occur for several months, some of them heavy. B) Normal activity will be permitted within 48 hours following surgery. C) After a hysterectomy, hormone levels remain largely unaffected. D) The bladder must be emptied prior to surgery and a catheter may be placed during surgery.

D. The intestinal tract and the bladder need to be empty before the patient is taken to the OR to prevent contamination and injury to the bladder or intestinal tract. The patient is informed that her periods are now over, but she may have a slightly bloody discharge for a few days. The patient is instructed to avoid straining, lifting, or driving until her surgeon permits her to resume these activities. The patient's hormonal balance is upset, which usually occurs in reproductive system disturbances. The patient may experience depression and heightened emotional sensitivity to people and situations.

Anaplastic: Cells lack normal characteristics, can differ in shapes and organization. Not reversible.

Degree of anaplasia determines risk for cancer

AKI output should be 30-50mL/hr MAINTAIN systolic BP >than 100

Electrical injury output should be 100mL/hr

Dysplastic: The cell becomes altered in size, shape, and organization. Cell starts to lose normal cell differentiation.

Ex: Cervical cells that are inflamed due to HPV

Metaplastic: Occurs in injury, irritation, or inflammation. Cell is still normal but location is not. Still normal DNA and is reversible.

Ex: Endometriosis

cranial nerve seven VII

Facial

Endocrine glands

Glands of the endocrine system that release hormones into the bloodstream

cranial nerve nine IX

Glossopharyngeal

Rule of 9's

Head and neck = 9% Upper Ex = 9% each Lower Ex = 9% each Front trunk = 18% Back trunk = 18%

cranial nerve twelve XII

Hypoglossal

Low levels of T3 and T4 Increase in levels of TSH

Hypothyroidism

Benign cells are well differentiated cells that resemble normal cells.

Malignant cells are undifferentiated and may not resemble a normal cell from its tissue origin.

RA- morning stiffness lasting more than 1 hour

OA- less than 30 minutes upon waking

Hyperplastic: increased number or density of normal cells

Occurs in response to stress, increased metabolic demand, increased level of hormones.

cranial nerve three III

Oculomotor

cranial nerve one I

Olfactory (smell)

cranial nerve two II

Optic - vision

Most common sx of Rheumatoid Disease:

PAIN along with Joint swelling Limited movement Stiffness Weakness Fatigue

Six P's of compartment syndrome

Pain Poikilothermia Pallor Pulselessness Paresthesia Paralysis

Causes of Acute Renal Failure

Prerenal Intrarenal Postrenal

Foods to avoid to prevent a goiter

Shellfish, any foods high in iodine

Bone healing stages

Stage I: hematoma (1 to 2 days) Stage II: Fibrocartilaginous callus formation (Fibroblasts/osteoblasts migrate to site of fx.) Stage III: Bony callus formation (ossification begins 3rd/4th weeks) Stage IV: Remodeling (osteoclasts move to necrotic bone) *Bone is in a constant state of turnover.*

Early this morning, a client had a subtotal thyroidectomy. During evening rounds, the nurse assesses the client, who now has nausea, a temperature of 105° F (40.5° C), tachycardia, and extreme restlessness. What is the most likely cause of these signs? a) Tetany b) Thyroid crisis c) Diabetic ketoacidosis d) Hypoglycemia

Thyroid crisis FEVER and HEART RATE Thyroid crisis usually occurs in the first 12 hours after thyroidectomy and causes exaggerated signs of hyperthyroidism, such as high fever, tachycardia, and extreme restlessness.

Fluid/electrolyte shifts in EMERGENT phase:

Trauma releases K+ into extracellular fluid- Hyperkalemia Na+ becomes trapped in edematous fluid- Hyponatremia Metabolic acidosis r/t massive tissue damage and release of cytokines.

cranial nerve five V

Trigeminal

cranial nerve four IV

Trochlear

cranial nerve ten

Vagus

Emergency department (ED) staff members have been trained to follow steps that will decrease the risk of secondary exposure to a chemical. When conducting decontamination, staff members should remove the patients clothing and then perform what action? A) Rinse the patient with water. B) Wash the patient with a dilute bleach solution. C) Wash the patient chlorhexidine. D) Rinse the patient with hydrogen peroxide.

a The first step in decontamination is removal of the patients clothing and jewelry and then rinsing the patient with water. This is usually followed by a wash with soap and water, not chlorhexidine, bleach, or hydrogen peroxide.

The client with polycystic kidney disease asks the nurse, "Will my kidneys ever function normally again?" The best response by the nurse is: a) "As the disease progresses, you will most likely require renal replacement therapy." b) "Draining of the cysts and antibiotic therapy will cure your disease." c) "Dietary changes can reverse the damage that has occurred in your kidneys." d) "Genetic testing will determine the best treatment for your condition."

a) "As the disease progresses, you will most likely require renal replacement therapy." There is no cure for polycystic kidney disease. Medical management includes therapies to control blood pressure, urinary tract infections, and pain. Renal replacement therapy is indicated as the kidneys fail.

A nurse is teaching a client with adrenal insufficiency about corticosteroids. Which statement by the client indicates a need for additional teaching? a) "I may stop taking this medication when I feel better." b) "I will eat lots of chicken and dairy products." c) "I will see my ophthalmologist regularly for a check-up." d) "I will avoid friends and family members who are sick."

a) "I may stop taking this medication when I feel better." The client requires additional teaching because he states that he may stop taking corticosteroids when he feels better. Corticosteroids should be gradually tapered by the physician. Tapering the corticosteroid allows the adrenal gland to gradually resume functioning.

A client with newly diagnosed renal cancer is questioning why detection was delayed. Which is the best response by the nurse? a) "Very few symptoms are associated with renal cancer." b) "You should have sought treatment earlier." c) "Squamous cell carcinomas do not present with detectable symptoms." d) "Painless gross hematuria is the first symptom in renal cancer."

a) "Very few symptoms are associated with renal cancer." Renal cancers rarely cause symptoms in the early stage. Tumors can become quite large before causing symptoms. Painless, gross hematuria is often the first symptom in renal cancer and does not present until later stages of the disease. Adenocarcinomas are the most common renal cancer (about 80%),whereas squamous cell renal cancers are rare. It is not therapeutic to place doubt or blame for delayed diagnosis.

Patient education regarding a fistulae or graft includes which of the following? Select all that apply. a) Check daily for thrill and bruit. b) No IV or blood pressure taken on extremity with dialysis access. c) Cleanse site b.i.d. d) Avoid compression of the site. e) No tight clothing.

a) Check daily for thrill and bruit. b) No IV or blood pressure taken on extremity with dialysis access. d) Avoid compression of the site. e) No tight clothing. The nurse teaches the patient with fistulae or grafts to check daily for a thrill and bruit. Further teaching includes avoiding compression of the site; not permitting blood to be drawn, an IV to be inserted, or blood pressure to be taken on the extremity with the dialysis access; not to wear tight clothing, carry bags or pocketbooks on that side, and not lie on or sleep on the area. The site is not cleansed unless it is being accessed for hemodialysis.

A client in chronic renal failure becomes confused and complains of abdominal cramping, racing heart rate, and numbness of the extremities. The nurse relates these symptoms to which of the following lab values? a) Hyperkalemia b) Elevated white blood cells c) Hypocalcemia d) Elevated urea levels

a) Hyperkalemia Hyperkalemia is the life-threatening effect of renal failure. The client can become apathetic; confused; and have abdominal cramping, dysrhythmias, nausea, muscle weakness, and numbness of the extremities. Symptoms of hypocalcemia are muscle twitching, irritability, and tetany. Elevation in urea levels can result in azotemia, which can be exhibited in fluid and electrolyte and/or acid-base imbalance. Elevation of WBCs is not indicated.

A nurse is reviewing the history of a client who is suspected of having glomerulonephritis. Which of the following would the nurse consider significant? a) Recent history of streptococcal infection b) History of osteoporosis c) Previous episode of acute pyelonephritis d) History of hyperparathyroidism

a) Recent history of streptococcal infection Glomerulonephritis can occur as a result of infections from group A beta-hemolytic streptococcal infections, bacterial endocarditis, or viral infections such as hepatitis B or C or human immunodeficiency virus (HIV). A history of hyperparathyroidism or osteoporosis would place the client at risk for developing renal calculi. A history of pyelonephritis would increase the client's risk for chronic pyelonephritis.

A nurse is assessing a client with hyperthyroidism. What findings should the nurse expect? a) Weight loss, nervousness, and tachycardia b) Weight gain, constipation, and lethargy c) Exophthalmos, diarrhea, and cold intolerance d) Diaphoresis, fever, and decreased sweating

a) Weight loss, nervousness, and tachycardia Hyper=increase in metabolic rate Hypo=decrease in metabolic rate Weight loss, nervousness, and tachycardia are signs of hyperthyroidism. Other signs of hyperthyroidism include exophthalmos, diaphoresis, fever, and diarrhea.

cranial nerve eight VIII

acoustic

Cardiac effects of hyperthyroidism include which of the following? a) Decreased systolic BP b) Palpitations c) Bradycardia d) Decreased pulse pressure

b) Palpitations Cardiac effects may include sinus tachycardia, increased pulse pressure, and palpitations.

Which findings should a nurse expect to assess in client with Hashimoto's thyroiditis? a) Weight loss, increased appetite, and hyperdefecation b) Weight gain, decreased appetite, and constipation c) Weight loss, increased urination, and increased thirst d) Weight gain, increased urination, and purplish-red striae

b) Weight gain, decreased appetite, and constipation Hashimoto's thyroiditis, an autoimmune disorder, is the most common cause of hypothyroidism. It's seen most frequently in women older than age 40. Signs and symptoms include weight gain, decreased appetite; constipation; lethargy; dry cool skin; brittle nails; coarse hair; muscle cramps; weakness; and sleep apnea. Hashimoto-slow (everything slows down d/t decreased metabolic rate).

Osteogenesis:

bone formation

The nurse analyzes a urinalysis report. He is aware that the presence of this substance in the urine indicates a blood level that exceeds the kidney's reabsorption capacity. Select the substance. a) Bicarbonate b) Sodium c) Glucose d) Creatinine

c) Glucose Glucose is usually filtered at the level of the glomerulus. It does not normally appear in the urine. Renal glycosuria occurs if the glucose in the blood exceeds the amount that is able to be reabsorbed. The other substances are normally excreted in the urine.

A patient with a history of hypothyroidism is admitted to the intensive care unit unconscious and with a temperature of 95.2ºF. A family member informs the nurse that the patient has not taken thyroid medication in over 2 months. What does the nurse suspect that these findings indicate? a) Syndrome of inappropriate antidiuretic hormone (SIADH) b) Diabetes insipidus c) Myxedema coma d) Thyroid storm

c) Myxedema coma Myxedema coma is a rare life-threatening condition. It is the decompensated state of severe hypothyroidism in which the patient is hypothermic and unconscious. This condition may develop with undiagnosed hypothyroidism and may be precipitated by infection or other systemic disease or by use of sedatives or opioid analgesic agents. Patients may also experience myxedema coma if they forget to take their thyroid replacement medication. (less)

Which glands regulate calcium and phosphorus metabolism? a) Thyroid b) Adrenal c) Parathyroid d) Pituitary

c) Parathyroid Parathormone (parathyroid hormone), the protein hormone produced by the parathyroid glands, regulates calcium and phosphorus metabolism.

Which is the most common motor dysfunction seen in clients diagnosed with stroke? a. Diplopia b. Ataxia c. Hemiparesis d. Hemiplegia

d Hemiplegia which is caused by a lesion of the opposite side of the brain. Motor dysfunction

A client with chronic renal failure comes to the clinic for a visit. During the visit, he complains of pruritus. Which suggestion by the nurse would be most appropriate? a) "Liberally apply alcohol to the areas of your skin where you itch the most." b) "When you shower, use really warm water and an antibacterial soap." c) "Try washing clothes with a strong detergent to ensure that all impurities are gone." d) "Keep your showers brief, patting your skin dry after showering."

d) "Keep your showers brief, patting your skin dry after showering." The client with pruritus needs to keep the skin clean and dry. The client should take brief showers with tepid water, pat the skin dry, use moisturizing lotions or creams, and avoid scratching. In addition, the client should use a mild laundry detergent to wash close and an extra rinse cycle to remove all detergent or add 1 tsp vinegar per quart of water to the rinse cycle to remove any detergent residue.

A 24-hour urine collection is scheduled to begin at 8:00 am. When should the nurse initiate the procedure? a) With the first specimen voided after 8:00 am b) At 8:00 am, with or without a specimen c) 6 hours after the urine is discarded d) After discarding the 8:00 am specimen

d) After discarding the 8:00 am specimen A 24-hour collection of urine is the primary test of renal clearance used to evaluate how well the kidney performs this important excretory function. The client is initially instructed to void and discard the urine. The collection bottle is marked with the time the client voided. Thereafter, all the urine is collected for the entire 24 hours. The last urine is voided at the same time the test originally began.

Accidental removal of one or both parathyroid glands can occur during a thyroidectomy. Which of the following is used to treat tetany? a) Propylthiouracil (PTU) b) Synthroid c) Tapazole d) Calcium gluconate

d) Calcium gluconate Sometimes in thyroid surgery, the parathyroid glands are removed, producing a disturbance in calcium metabolism. Tetany is usually treated with IV calcium gluconate.

Which of the following disorders is characterized by a group of symptoms produced by an excess of free circulating cortisol from the adrenal cortex? a) Graves' disease b) Addison's disease c) Hashimoto's disease d) Cushing syndrome

d) Cushing syndrome The patient with Cushing syndrome demonstrates truncal obesity, moon face, acne, abdominal striae, and hypertension. Regardless of the cause, the normal feedback mechanisms that control the function of the adrenal cortex become ineffective, and the usual diurnal pattern of cortisol is lost. The signs and symptoms of Cushing syndrome are primarily a result of the oversecretion of glucocorticoids and androgens, although mineralocorticoid secretion also may be affected.

The nurse notes that the client's urine is blood-tinged following cystoscopy. Which of the following nursing actions should the nurse do next? a) Instruct the client to increase fluid intake. b) Inspect the client's urinary meatus. c) Notify the physician of the finding. d) Document the finding in the health record.

d) Document the finding in the health record. The physician does not need to be contacted as blood-tinged urine is an expected finding following cystoscopy due to trauma of the procedure. The nurse should document the finding and continue to monitor the client. The client should be encouraged to increase fluid intake to help flush the urinary tract of microorganisms. The urinary meatus does not need to be inspected.

The nurse is reviewing a client's history which reveals that the client has had an oversecretion of growth hormone (GH) that occurred before puberty. The nurse interprets this as which of the following? a) Acromegaly b) Simmonds' disease c) Dwarfism d) Gigantism

d) Gigantism When over secretion of GH occurs before puberty, gigantism results; before the fusion of the epiphyseal plates. Whereas Acromegaly occurs after the fusion.

The nurse is passing out medications on a medical-surgical unit. A male patient is preparing for hemodialysis. The patient is ordered to receive numerous medications including antihypertensives. Which of the following is the best action for the nurse to take? a) Administer the medications as ordered. b) Ask the patient if he wants to take his medications. c) Check with the dialysis nurse about the medications. d) Hold the medications until after dialysis.

d) Hold the medications until after dialysis.

On the third day after a partial thyroidectomy, a client exhibits muscle twitching and hyperirritability of the nervous system. When questioned, the client reports numbness and tingling of the mouth and fingertips. Suspecting a life-threatening electrolyte disturbance, the nurse notifies the surgeon immediately. Which electrolyte disturbance most commonly follows thyroid surgery? a) Hypermagnesemia b) Hyperkalemia c) Hyponatremia d) Hypocalcemia

d) Hypocalcemia Hypocalcemia may follow thyroid surgery if the parathyroid glands were removed accidentally. Signs and symptoms of hypocalcemia may be delayed for up to 7 days after surgery.

A nurse is preparing to palpate the thyroid gland. Where would the nurse expect to find this gland? a) In the abdomen, directly above the kidneys b) In the upper part of the chest near the heart c) In the right to left upper quadrant of the abdomen d) In the lower neck, anterior to the trachea

d) In the lower neck, anterior to the trachea The thyroid gland is located in the lower neck, anterior to the trachea.

Which of the following glands is considered the master gland? a) Parathyroid b) Adrenal c) Thyroid d) Pituitary

d) Pituitary secretes ACTH and the adrenal cortex secretes cortisol.

Retention of which electrolyte is the most life-threatening effect of renal failure? a) Calcium b) Phosphorous c) Sodium d) Potassium

d) Potassium Retention of potassium is the most life-threatening effect of renal failure.

A nurse is assessing a client after a thyroidectomy. The assessment reveals muscle twitching and tingling, along with numbness in the fingers, toes, and mouth area. The nurse should suspect which complication? a) Laryngeal nerve damage b) Hemorrhage c) Thyroid storm d) Tetany

d) Tetany Tetany may result if the parathyroid glands are excised or damaged during thyroid surgery.

A client with renal dysfunction of acute onset comes to the emergency department complaining of fatigue, oliguria, and coffee-colored urine. When obtaining the client's history to check for significant findings, the nurse should ask about: a) childhood asthma. b) chronic, excessive acetaminophen use. c) family history of pernicious anemia. d) recent streptococcal infection.

d) recent streptococcal infection. A skin or upper respiratory infection of streptococcal origin may lead to acute glomerulonephritis. Other infections that may be linked to renal dysfunction include infectious mononucleosis, mumps, measles, and cytomegalovirus. Chronic, excessive acetaminophen use isn't nephrotoxic, although it may be hepatotoxic. Childhood asthma and a family history of pernicious anemia aren't significant history findings for a client with renal dysfunction.

Role of Calcium:

decrease calcium and vitamin D leading to brittle bones

Diarthrosis

freely movable joint

Synarthrosis

immovable joint

Thyroid Storm

increased temp, pulse and HTN

LR is choice for emergent/rescuitative because:

of its pH, osmolarity and similar albumin components

Rheumatoid Arthritis (RA) risk factors

pollution, smoking, family hx, bacterial/viral infections

Ampiarthrosis

slightly movable joint

cranial nerve eleven XI

spinal accessory


Kaugnay na mga set ng pag-aaral

Tort Ch. 12 Special Tort Actions

View Set

Muscles of the Arm and Forearm: Origin, Insertion, and Action

View Set

EOSC 118: Lesson 7 - Minerals: Parent Gem Minerals

View Set

Vista previa- Realidades 3-Vocabulario en contexto-Pp. 68-71

View Set

chapter 16 - dilutive securities and earnings per share

View Set